Космос и астрономия


Ответить в тред Ответить в тред

Check this out!
<<
Назад | Вниз | Каталог | Обновить тред | Автообновление
541 49 183

Тред тупых вопросов №103 EINSTEIN EDITION Аноним 25/02/19 Пнд 13:58:47 4644881
th.jpg (21Кб, 474x648)
474x648
Einsteincross.jpg (43Кб, 800x772)
800x772
dsc5187.jpg (230Кб, 863x574)
863x574
WarpedSpaceandT[...].webm (9163Кб, 854x480, 00:01:13)
854x480
Тред вопросов о жизни, Вселенной и всём таком.

Спрашиваем то, за что в других местах выдают путёвку в биореактор. Здесь анонимные учёные мирового уровня критически рассмотрят любые гениальные идеи и нарисованные в Paint схемы.

Предыдущий тут: >>459163 (OP)
https://2ch.hk/spc/res/459163.html

Q: Можно быстрее?
A: Можно упасть в пузырь альбукерке, наса уже почти надула его.

Q: Я начитался охуительных историй про уфологию, че делать, нам жопа?
A: Да, тебе жопа, можешь сгонять в зогач или куда оттуда пошлют.

Q: Что будет с человеком в вакууме без скафандра / если он упадет на черную дыру / попробует ступить на поверхность газового гиганта/солнца?
A: Он умрёт.

Q: Почему бы не привязать ракету к воздушному шару или стартовать с горы?
A: Космос - это не как высоко, а как быстро, большая часть энергии ракеты уходит на разгон вбок.
Подробнее тут https://what-if.xkcd.com/58/ (английский) https://chtoes.li/orbital-speed/ (перевод)
Аноним 25/02/19 Пнд 14:15:18 4644902
Einstein19211.png (131Кб, 486x605)
486x605
>>464488 (OP)
Ебанный стыд...
Во-первых, Алькубьерре.
Во-вторых, не упасть, а создавать вокруг корабля изнутри (иначе кина не будет).
В-третьих, НАСА искривляет пространство на десятимиллионную часть, контролируя это сверхточными интерферометрами, до самого варп-привода здесь - как до Антарктиды раком.
Аноним 25/02/19 Пнд 14:17:00 4644913
>>464487
Подожди, так разве это не обычная теория гравитации с гравитонами, просто с заменой знака на противоположный? Этакое море Дирака, гравитация эдишен.

Ну, как если бы мы рассуждали об электромагнитном взаимодействии, просто вместо электрона как базовой частицы взяли бы дырку — его отсутствие. Тогда бы, например, при рассмотрении атома у нас бы вместо окружающего вакуума получилась заполненная дырками внешняя среда, а атом вместо электронных облаков имел бы поля разрежения. И так далее.

Тогда, конечно, все предсказания будут выполняться и все уравнения совпадут один в один, просто непонятно, а в чем смысл такой теории?
Аноним 25/02/19 Пнд 14:21:54 4644924
>>464488 (OP)
>WarpedSpaceandT[...].webm
Это что при слиянии черных дыр образуется антигравитация?
Аноним 25/02/19 Пнд 14:22:57 4644935
>>464491
В том-то и дело, что "альтернативы" ОТО либо оказываются ОТО in disguise, либо проебываются по воспроизводству ее предсказаний.
Аноним 25/02/19 Пнд 14:40:02 4644946
>>464491
>Подожди, так разве это не обычная теория гравитации с гравитонами, просто с заменой знака на противоположный?

Нет, как до этого анон написал почти чисто механистическая теория.

И это не гравитоны, мне само название не нравится, ведь оно ассоциируется с откровенной глупостью - кинул шарик в груду других шариков, а они с какого-то хрена притянулись, а не получили импульс.

Здесь идея именно в том, что есть так называемый первород, из которого состоят протон и протон со свёрнутым полем.
Электрон это присоединённый вихрь из газовой динамики. То, что мы наблюдаем это грубо говоря осколки, ну или группа, большая учитывая их размеры, которая обладая изначальными скоростями собралась такой же, но гораздо менее долго живущий вихрь. По предположениям протоны живут около 10 миллиардов лет, дальше распадаются. То, что в центре галактики создаёт протоны новые, оно по теории может быть вечно, если ещё одно такое не объявится рядом.

>в чем смысл такой теории?

В том, что сразу становится ясно как создать двигатель, который позволит летать к звёздам - по этой теории, внутри тел с гравитационным полем крайне слабо или даже совсем не действуют внешние силы гравитации и сама инерция, ведь её природа кроется в самой сути этой среды, получается такой же эффект, который нам показывают изображая ЧД, окружающее пространство как бы обтекает такой вихрь и слабо мешает движению. То есть если резко остановить Землю, люди с неё не улетят, они даже не заметят этого. Теперь к движку, тоже самое, любые ускорения не будут влиять на тех кто внутри, получается за одну секунду можно будет долететь до плутона или до другого края галактики и даже в другую галактику.


Аноним 25/02/19 Пнд 14:53:27 4644957
>>464494
Для мелких частиц должны быть охуенно сильные квантовые эффекты.
>протоны живут около 10 миллиардов лет, дальше распадаются
Что несешь, вообще ахуеть.
>если резко остановить Землю, люди с неё не улетят, они даже не заметят этого
ЧЕЕЕЕЕЕЕЕЕЕЕЕЕЕЕЕЛ!
Аноним 25/02/19 Пнд 15:08:18 4644968
>>464495
>>если резко остановить Землю, люди с неё не улетят, они даже не заметят этого
>ЧЕЕЕЕЕЕЕЕЕЕЕЕЕЕЕЕЛ!
что не так блять?
25/02/19 Пнд 15:24:15 4644979
>>464496
>30 км/с орбитального движения
>резко остановить
>не улетят
Аноним 25/02/19 Пнд 15:46:13 46449810
Аноним 25/02/19 Пнд 16:01:38 46450111
>>464495
>Для мелких частиц должны быть охуенно сильные квантовые эффекты.

Они и должны наблюдаться, ведь для них влияние этой среды будет сильнее.

Касаемо Земли, опять же выше я привёл объяснение уровня 17 века, ну может 18.
Аноним 25/02/19 Пнд 16:07:31 46450412
>>464488 (OP)
С какой высоты начинается космос?
Есть ли чёткая граница атмосферы?
На какой предельной высоте удерживаются гравитацией атомы воздуха?
Аноним 25/02/19 Пнд 16:10:57 46450713
>>464486
>Вся суть механистического объяснения как раз в экранировании телами друг друга от давления внешней среды.
Если клепать механистичную теорию гравитации, то можно отождествить ее с температурой в "гравитонном" газе, типа холодные тела приталкиваются в горячем газе. Экранирования при этом нет, просто градиенты температур вокруг тел накладываются друг на друга.

>Эксперименты по измерению скорости гравитационных волн показывают, что они распространяются со скоростью света, так что тоже не катит.
Скорее всего это не точные эксперименты. Механическую теорию гравитации невозможно построить без привлечения сверхсветовых скоростей, что дает основание в сомнении к таким результатам экспериментов. Да и гравитационные волны исходя из такого подхода представляют собой волны температуры, которые очень быстро затухают в среде, то есть в эксперименте возможно измеряли не скорость гравитационных волн, а нечто другое.
Аноним 25/02/19 Пнд 16:13:36 46450814
>>464504
100 км
не очень
высоко, вплоть до тысяч км
Аноним 25/02/19 Пнд 16:14:55 46450915
>>464488 (OP)
> пик 2
Почему именно четыре ярких источника, а не смазанное кольцо?
Аноним 25/02/19 Пнд 16:53:37 46451216
>>464508
Спсбо, ну т.е. не один атом воздуха не улетит от нас.
А то я чёт переживал, что через озоновые дыры всё
сдрыснет в холодный вакуум и подзадохнёмся тута.
Аноним 25/02/19 Пнд 17:17:03 46451417
>>464507
>Скорее всего это не точные эксперименты. Механическую теорию гравитации невозможно построить без привлечения сверхсветовых скоростей, что дает основание в сомнении к таким результатам экспериментов.

Да просто механистическая теория говно нерабочее, вот и все.

>то есть в эксперименте возможно измеряли не скорость гравитационных волн, а нечто другое.

https://ru.wikipedia.org/wiki/GW170817 — вот это наблюдение, про которое я говорил. В течение 2 секунд в одной области неба произошел гамма-всплеск и из того же самого места был зафиксирован приход гравитационных волн, при этом соответствующий по амплитуде наблюдениям. Как-то уж совсем маловероятно для совпадения.

Сверхточные интерферометры, те самые из копипасты, меряют изменение метрики пространства, сложно представить, что на их показания могло повлиять что-то помимо гравитационных волн.
Аноним 25/02/19 Пнд 18:40:09 46452118
>>464512
Ну, в общем так и есть, Земляшка постепенно теряет атмосферу, но до превращения Солнца в красного гиганта нам ее скорее всего хватит.
Аноним 25/02/19 Пнд 18:43:32 46452219
>>464514
Нет, сама идея изменения расстояния в некоторых системах говно нерабочее и вызывает только смех. То что там они измеряют может быть что угодно.

Касаемо наблюдения. Амплитуда чего? Гамма излучения, которая подобна или равна гравитационной?
Аноним 25/02/19 Пнд 18:44:20 46452320
>>464521
>Земляшка постепенно теряет атмосферу
Да, но при этом что-то попердывают вулканы, то есть выходят газики, растворенные в мантии. Какой там баланс выходит - хз. К тому же, теряются в основном легкие водород и гелий, которые один хуй никому не нужны.
Аноним 25/02/19 Пнд 18:45:09 46452421
>>464512
Воздух держится у нас гравитацией, и ещё магнитное поле планеты защищает от того, чтобы его солнечным ветром не сдуло. Озоновые дыры это немного другое, озон это тоже газ, он просто солнечный ультрафиолет до поверхности не пропускает.

Но атмосфера всё равно потихоньку теряется, однако так медленно, что нам нечего бояться. Тебе на жизнь хватит.
25/02/19 Пнд 19:02:42 46452522
>>464522
Иди нахуй, говно.
Аноним 25/02/19 Пнд 19:09:28 46452623
>>464525

Ожидаемо, но ты не тот анон, что мне отвечал на вопросы, ты тупее, если тебе настолько неприятна эта тема, то игнорируй. Напоминаю, это тред тупых вопросов. Или тут уже успели появиться запретные темы? Ясное дело, что отвечать на вопросы уровня а чё ето атомы не улетаю с Земли проще, чем разъяснить в чём я не прав.
Аноним 25/02/19 Пнд 20:57:56 46454824
>>464508
>100 км
Скоро на 80 км опустят - для НАСА и ВВС США - космос уже давно начинается на 80 км.
Фактически космос начинается там - где воздух не оказывает существенного влияния на орбиту, то есть примерно с 80 км.
Сам Карман установил границу на высоте 83,9 км - но для него это была высота, куда может долететь самолет - используя подъемную силу.
Есть ещё и военная составляющая - ибо космос общий, а воздушное пространство до космоса принадлежит отдельным странам. Чем выше космос - тем спокойнее.
https://www.fai.org/news/statement-about-karman-line
25/02/19 Пнд 22:07:32 46458325
>>464526
Ну что ж ты так. За добавкой вернулса маня?
Аноним 26/02/19 Втр 11:19:28 46465626
>>464583
>Мам, пространство может искривляться и похуй, что это пустота
>Мам, быстрее света ничего двигаться не может и похуй, что это выведено из преобразований Лоренца

Вот тебе добавки. Странно, что вообще сейчас есть люди, что всерьёз в эти бредни верят, именно верят, ведь здравый смысл это отметает.

Ну и напоследок, почитай, что про все эти бредни писал Циолковский.
Аноним 26/02/19 Втр 11:49:07 46466727
Чем запуск отличается от пуска?
Аноним 26/02/19 Втр 13:56:55 46471628
>>464667
Тем же, чем заебенить отличается от ебенить.
26/02/19 Втр 14:20:40 46473429
>>464656
>здравый смысл это отметает
>что про все эти бредни писал Циолковский
Уходи. Нахуй.
Аноним 26/02/19 Втр 14:24:15 46473630
>>464656
А где на дваче можно пообсуждать эфир в кругу единомышленников, без тролльских эйнштенодрочеров?
Аноним 26/02/19 Втр 14:56:36 46476131
Аноним 26/02/19 Втр 16:09:09 46477732
>>464736
Отдельный тред создать, если действительно много есть что пообсуждать, а если дискаса мало, то можно прямо тут.
26/02/19 Втр 16:24:48 46477833
>>464777
>заговорчики в /zog/
Аноним 26/02/19 Втр 17:00:20 46478234
>>464778
https://wikilivres.org/wiki/Äther_und_Relativitätstheorie
>Nach der allgemeinen Relativitätstheorie ist der Raum mit physikalischen Qualitäten ausgestattet; es existiert also in diesem Sinne ein Äther. Gemäß der allgemeinen Relativitätstheorie ist ein Raum ohne Äther undenkbar; denn in einem solchen gäbe es nicht nur keine Lichtfortpflanzung, sondern auch keine Existenzmöglichkeit von Maßstäben und Uhren, also auch keine räumlich-zeitlichen Entfernungen im Sinne der Physik.

Перевод:
Общая теория относительности наделяет пространство физическими свойствами; таким образом, в этом смысле эфир существует. Согласно общей теории относительности, пространство немыслимо без эфира; поскольку в таком пространстве не только было бы невозможно распространение света, но и не могли бы существовать масштабы и часы и не было бы никаких пространственно-временных расстояний в физическом смысле слова.

Даже Эйнштейн не против был эфира, говорил что его теория эфирная, просто модель эфира несколько иная. Так почему бы не обсуждать остальные эфирные модели если эфир есть?
Аноним 26/02/19 Втр 17:34:52 46478735
>>464782
>Так почему бы не обсуждать остальные эфирные модели если эфир есть?
Наконец-то шизик созрел для Тупого Вопроса. Ответ на него очевиден остальные эфирные модели надежно опровергаются результатами наблюдений и экспериментов, но шизик продолжает делать вид, что 100+ лет релятивизма — это на самом деле в говне моченые все подстроили, ага. С этим лучше в /zog.
Аноним 26/02/19 Втр 17:49:32 46478836
Предположим, что мы покрываем Меркурий тонким слоем вещества с очень высокой отражательной способностью и по-тихоньку ждём, пока его орбита растянется до орбиты Венеры. Насколько сложно будет сделать его спутником Венеры? Сколько придётся ждать и сработает ли это вообще? Есть ли какие-нибудь более эффективные способы?
Аноним 26/02/19 Втр 18:04:16 46479337
>>464788
С чего ты взял, что это вообще будет иметь какой-то эффект? Максимум - это увеличение импульса от светового давления в два раза (полное отражение по сравнению с полным поглощением).
Аноним 26/02/19 Втр 18:05:26 46479438
>>464793
Ну не ракетами же его двигать. Первое что пришло в голову из околоосуществимого.
Аноним 26/02/19 Втр 18:09:36 46479739
>>464794
Если бы Солнце еще светило Меркурию в жопу, а не в бок по направление движения (что, понятно, невозможно в системе, где Меркурий вращается вокруг Солнца), что-то может и вышло бы. Хотя подозреваю, что это световое давление настолько ничтожно, что и в этом случае ничего бы не дало.
Аноним 26/02/19 Втр 18:15:19 46479840
>>464794
Значит, укрепляем поверхность, ставим огроменное пихло и потихоньку начинаем ускорять планету выбирая нужные моменты для запуска. Я, правда, даже хуй знает что сложнее, построить и притащить топливо для такой йобы или обмазать весь шарик блестяшками.
Аноним 26/02/19 Втр 18:19:41 46479941
4e3f655a6fc421.png (538Кб, 854x484)
854x484
>>464488 (OP)
Я правильно понимаю, что в этих половых губах на скриншоте из вебм кратковременно возникает отрицательная кривизна пространства, и скорость света там больше обычной С?
Аноним 26/02/19 Втр 18:47:12 46480442
>>464799
Почему ты думаешь, что она там больше с?
Аноним 26/02/19 Втр 18:57:15 46480943
>>464787
Из чего состоит бозон и из чего состоит это пространство? М?
Аноним 26/02/19 Втр 19:00:55 46481244
Сап, спейсаны. Возможно вопрос не совсем по адресу, но всё же.
Собственно, наука уже выяснила почему скорость света именно такая какая она есть а не другая?
Аноним 26/02/19 Втр 19:27:39 46481445
>>464787
>остальные эфирные модели надежно опровергаются результатами наблюдений и экспериментов
Слишком самоуверенное заявление.
Аноним 26/02/19 Втр 19:27:52 46481546
>>464804
Потому что там должно происходить нечто обратное гравитационному замедлению времени. Если провал подразумевает гравитационную яму с замедлением времени, а бездна под горизонтом событий - область, где время с точки зрения внешней вселенной останавливается, то в пучности время должно идти быстрее внешней вселенной. Ну и местная скорость света должна быть больше С.
Аноним 26/02/19 Втр 19:30:03 46481647
>>464812
Не.
Наука даже до сих пор не выяснила, почему число 8 именно такое, а не другое. Не говоря уж о всяких там числах пи...
Аноним 26/02/19 Втр 19:49:17 46481948
>>464488 (OP)
Как думаете может России отказаться вообще от космических программ, все равно нихуя не выходит, а так хоть деньги лишние будут.
Аноним 26/02/19 Втр 19:51:20 46482149
>>464816
Сейчас бы в 2к19 не понимать разницы между математическими абстракциями (8 и пи) и физическими константами (скорость света).
Аноним 26/02/19 Втр 20:22:46 46482450
>>464819
> а так хоть деньги лишние будут.
Разворуют.
Аноним 26/02/19 Втр 21:06:52 46482951
>>464812
>Собственно, наука уже выяснила почему скорость света именно такая какая она есть а не другая?
Конечно, скорость света ограниченна временем взаимодействия частиц эфира при передаче импульса, который ошибочно называют фотоном, на основании того факта, что конечная частица при передачи импульса через эфир выступает в роли фотона.
Аноним 26/02/19 Втр 21:14:33 46483252
>>464815
>должно
Ясно
Дальше этого дебила не читал.
Аноним 26/02/19 Втр 21:14:37 46483353
>>464812
В естественной системе единиц скорость света равна единице, так как у фотона нулевая масса покоя. Ты неправильно формулируешь вопрос. Правильный вопрос - не почему свет движется с такой скоростью, а почему у других частиц есть массы покоя, не позволяющие им двигаться с единичной скоростью. Квантовая теория поля объясняет массы частиц (кроме нейтрино) через взаимодействие с бозоном Хиггса. Однако почему они взаимодействуют с ним именно так, а не иначе - сия тайна велика есть.
Аноним 26/02/19 Втр 21:15:25 46483554
>>464829
>этот протекший из обо/сцайной параши малолетний шизодебил
Понятно.
Аноним 26/02/19 Втр 21:19:41 46483755
Dirac.jpg (34Кб, 450x610)
450x610
>>464835
Почему обосцай скатился в такое говно?
Аноним 26/02/19 Втр 21:22:59 46483956
>>464837
Всегда был парашей для говна и шизиков, со времен закрепленного треда где есть мой пост, хаха уже полностью.
Аноним 26/02/19 Втр 21:24:48 46484257
>>464839
Вопрос по теме закрепленного треда: хуи сосешь?
Аноним 26/02/19 Втр 21:26:37 46484458
Аноним 26/02/19 Втр 21:31:20 46485059
card 027 Jennif[...].png (618Кб, 539x800)
539x800
>>464844
Мод обосцача - тян?
Аноним 26/02/19 Втр 21:33:40 46485360
>>464850
Сейчас уже в любом случае нет.
Теперь в моче остались только школодебилы из самых червепидорских здесь тоже, кстати
Аноним 26/02/19 Втр 21:40:07 46485761
>>464853
Мочу вроде несколько раз выселяли же отсюда и сразу начинался вайп говном и спам антисетипетухами?
Аноним 26/02/19 Втр 22:33:50 46487462
>>464832
Если ты такой умный, объясни! Тебе сложна что ли?
Аноним 26/02/19 Втр 23:01:41 46488063
>>464833
Давай проще, на перестроение частиц эфира требуется некоторое время, поэтому импульс через плотноприжатые сферообразные частицы эфира не может распространяться мгновенно.
Аноним 27/02/19 Срд 00:28:01 46489364
>>464874
Деньги вперед.
>>464880
Да ты, холоп, не уймешься.
Аноним 27/02/19 Срд 00:28:39 46489465
>>464893
>Деньги вперед.

В светоносную среду веруешь?
Аноним 27/02/19 Срд 00:40:00 46489766
>>464894
Где деньги, маня?
Аноним 27/02/19 Срд 00:58:56 46489867
>>464897
Их нет, да я и сам разберусь, безплатно.

Время в этих участках почти безконечно ускорилось, из чего следует, что либо два этих шарика помереть должны были, испариться, либо потерять как минимум половину массы, пока это ускорение не прошло.

Вывод: слияние двух и более ЧД может привести к их полному испарению.
Аноним 27/02/19 Срд 01:35:52 46490168
>>464898
>Их нет
Дальше читать нищего шизодебила смысла нет.
Аноним 27/02/19 Срд 09:50:20 46492469
>>464894
>В светоносную среду веруешь?
а что лучше в мистические поля веровать, которые как бы есть, но которых и нет. это же уровень детсада.
а наличие светоносной среды в виде плотноприжатых сфер многое объясняет. например инерцию, когда сферы с фронта движения перетекают в тыл и тем самым поддерживают движение тела и тд.
Аноним 27/02/19 Срд 10:45:08 46493270
>>464924
про гравитационные волны что расскажешь?
Аноним 27/02/19 Срд 10:58:34 46493371
>>464901
О-объективность.

А зачем ты тогда нужен, если по факту ничего объяснить не способен? Да, вот они, учёные 21 века!

>>464924
Чем тебе прост газ не угодил, то? Тут выше анон отличную идею про быстрое затухание тепловых волн предложил, ща изучать буду.
Про инерцию согласен, что эта самая среда её и создаёт и поддерживает.
Аноним 27/02/19 Срд 11:20:01 46493572
>>464932
>про гравитационные волны что расскажешь?
эфирная среда и порождает гравитацию, а волна в среде это типичное явление
>>464933
>Чем тебе прост газ не угодил, то?
пустота невозможна, размерностью может обладать только материальное тело. соответственно всё пространство заполнено материей без образования пустоты. но такое пространство будет подобно бетону без возможности к движению. однако частицы материи без осевого вращения могут свободно делиться и объединяться, образуя тем самым идеальную жидкость, которая оптимальным образом подстраивается под промежутки между плотноприжатыми сферами, не допуская образования пустоты. солнце является буфером идеальной жидкости в центре вихря из плотноприжатых сфер.
Аноним 27/02/19 Срд 11:37:00 46493773
>>464933
>>464935
Два дебила - это сила.
Обо/сцай на выезде, хаха.
А разгадка проста: весна, шизики обостряются
Аноним 27/02/19 Срд 11:54:37 46494174
>>464935
Эта среда как жидкость уже не раз терпела провал, беда в этом. Там вихри или не могут исчезнуть или не могут образоваться.

>>464937
Если ты действительно учёный, то это печально, ведь твоя задача не обезьянничать, а указать на ошибку.
Аноним 27/02/19 Срд 12:09:58 46494275
>>464941
>указать на ошибку.
Очевидно же что ошибка у тебя в количестве хромосом
Аноним 27/02/19 Срд 12:11:23 46494376
>>464942
Жаль, что ты не способен на конструктивную критику, поэтому то и у нас потешная космическая эра.
Аноним 27/02/19 Срд 12:17:34 46494477
>>464943
>поэтому то и
>потешная
Оно еще и деревенское.
Впрочем, ничего нового.
Аноним 27/02/19 Срд 12:29:48 46494578
>>464944

Хех, знаешь, ты мне напомнил одного долбоёба, что доказывал так называемую закопанность домов - звёздной пылью, я не шучу, это оно дейсвтительно изрыгало из своего рта, выбросом мусора и его не уборкой, в деревнях. Этот долбоёб учёный типа кстати и так же обо всех неумных отзывался.

Эх, благодарю тебя за чёткое понимание полного и окончательного краха как системы образования, так и науки.
Аноним 27/02/19 Срд 12:34:20 46494679
>>464945
>дейсвтительно
А ручонки-то дрожат.
>выбросом мусора и его не уборкой, в деревнях.
>эта ОБИДА копротивляющегося деревенского быдла
Ясно.
Аноним 27/02/19 Срд 12:34:33 46494780
>>464945
Ну если разбираться, то так оно и есть. Все химические элементы кроме H это звёздная пыль, по сути.
Аноним 27/02/19 Срд 12:37:30 46494881
>>464946
С чего ты взял, что я на тебя обиделся? Я прост жду, когда ты мне ответишь, хотя бы на невозможность образования тороидального вихря в газе с более меньшим трением частиц, вязкостью и так далее.

>>464947
Это понятно, но объяснять подобным образом данное явление это заявка на безумие.
Аноним 27/02/19 Срд 12:39:24 46495082
>>464941
>Эта среда как жидкость уже не раз терпела провал, беда в этом. Там вихри или не могут исчезнуть или не могут образоваться.
хрень, такая теория даже никогда и не обсуждалась, хотя и спизжена у декарта.
вихри образуются из плотноприжатых сфер, идеальная жидкость тупо заполняет промежутки. правда когда идеальная жидкость теряет скорость, то она превращается в частицу и если эта частица приобретет осевое вращение (спин), то она не будет разрушена. так рождаются всякие спиралевидные и прочие элементарные частицы.
Аноним 27/02/19 Срд 12:43:10 46495183
>>464948
>тороидального вихря в газе
ацюковский это же шарлатан, что у него находится в промежутках между частицами газа? пустота? но она невозможна! а если всё пространство заполнить частицами газа без образования пустоты, то мы получим бетон в котором движение невозможно.
Аноним 27/02/19 Срд 12:46:16 46495284
>>464951
>что у него находится в промежутках между частицами газа

Это такое же предположение, как и постоянство скорости света, ничего более, и потом уже от этого шагаю остальные предположения. Что между ними сложно ответить.

Моё мнение, что её нужно проверить, я размышляю над этим.
Аноним 27/02/19 Срд 12:49:28 46495485
>>464948
>эти оправдания деревенского омегана
Ясно.
>>464950
>>464951
>эти беседы с залетным обо/сцайным шизиком
Понятно.
>>464952
>я размышляю
Лучшая шутка месяца.
Аноним 27/02/19 Срд 12:50:38 46495686
>>464954
Почему ты такой злой и агрессивный? Тебя кто-то обидел?
Аноним 27/02/19 Срд 12:51:17 46495787
>>464952
>Это такое же предположение, как и постоянство скорости света, ничего более, и потом уже от этого шагаю остальные предположения.
а вот это главная блядь ошибка. вы сперва логичную модель эфира постройте, а потом шагайте дальше. а то хренота получается: "мы сперва допустим существование бреда, а потом от него шагать будем"
надеюсь, ты понимаешь, что газами никогда пространство на 100% не заполнить, иначе это будут уже не газы, а пустота невозможна. ну не может быть пустого пространства. как "ничто" может обладать размерностью?
Аноним 27/02/19 Срд 12:52:50 46495888
>>464956
>эти риторические вопросы
Верный признак и симптом лоха по жизни, кстати.
Аноним 27/02/19 Срд 12:53:23 46495989
>>464957
>этот мелкобуквенный шизик
Они на свет лезут.
Аноним 27/02/19 Срд 12:54:22 46496090
>>464958
>я обтекаю
Сливаться ты умеешь, пиздуй по холодку.
Аноним 27/02/19 Срд 12:56:40 46496191
>>464960
>эта ОБИДА НА ПРАВДУ
As expected.
Аноним 27/02/19 Срд 13:00:17 46496292
>>464957

Забыл, утверждается, что пространство безконечно и так же газ его безконечно заполняет. Знаю, нарушение закона сохранения энергии, но изначально твоё требование удивляет, ведь и та же теория большого взрыва имеет подобного рода проблемы.
Аноним 27/02/19 Срд 13:01:34 46496393
>>464962
>безконечно
>безконечно
Ох уж это деревенское быдло.
Аноним 27/02/19 Срд 13:04:44 46496594
>>464963
Стопе, ты с антропогеза что ли? Я тебя задел? Да? Ну что поделать, не нравитесь вы мне, я бы ни не ругался, если бы не подход к доказательствам и опровержениям, которые даже хуже чем это могут сделать пятиклашки.
Аноним 27/02/19 Срд 13:09:14 46496695
>>464965
>?
>?
>?
Ебать ты чмо омежное по жизни.
Аноним 27/02/19 Срд 13:11:54 46496796
>>464962
>Забыл, утверждается, что пространство безконечно и так же газ его безконечно заполняет
как может существовать газ без образования пустоты? ацюковский шарлатан, че его слушать.
я знаю его бредни между крупным газом более мелкий газ и так до бесконечности, но вся правда кроется в том, что для того чтобы пододвинуть частицу самого крупного газа, нам нужно бесконечно много перестроений более мелких, а это займет бесконечно много времени.
опять же из той теории ацюковского следует, что он допускает пустоту в бесконечности, а если её нет, то у него бетон, а не газы.
>ведь и та же теория большого взрыва имеет подобного рода проблемы
а это вообще бредовая гипотеза!
материя не рождается и не исчезает - она переходит из одной формы в другую. ошибка даунов, которые выдумали ТБВ в том, что у них энергия может существовать отдельно от материи, а это невозможно! энергия это мера движения материи, энергия без материи невозможна.

Аноним 27/02/19 Срд 13:12:43 46496897
>>464966
То что меня тут забанят это был вопрос времени, смешно, ты действуешь как омежка и бежишь жаловаться маме, вместо того чтобы ответить по делу. И при этом ты первым стал на меня нарываться и совершенно игнорировать мои вопросы, только оскорбления в ответ.

Зарепортил в ответ за щитпостинг.
Аноним 27/02/19 Срд 13:15:37 46496998
>>464968
> И при этом ты первым стал на меня нарываться и совершенно игнорировать мои вопросы, только оскорбления в ответ
а ему нечего ответить. в официальной физике такие бредовые теоретические выкладки, что они не выдерживают никакой критики. если бы он с нами вступил в спор, то это было бы аналогично томы, что он бы снял штаны и нагнулся.
Аноним 27/02/19 Срд 13:15:39 46497099
>>464968
>этот спалившийся дегенерат с обо/сцая
Надо же, эта гнида и правда оттуда, хаха.
>ответить по делу
Тут как нельзя более уместны золотые, бессмертны слова: соси хуй, быдло
Аноним 27/02/19 Срд 13:16:42 464971100
>>464969
>он с нами вступил в спор
Споры - они в реальной ИРЛ жизни, хикка мамина.
Но там ты молчишь и глазки прячешь, хаха.
Аноним 27/02/19 Срд 13:19:23 464972101
>>464967
Так и с жидкостью и с полями всё та же проблема, откуда оно взялось вообще и из чего состоит.

Любопытно, что мы снова приходим к дифференциалам и интегралам, а именно к безконечному приращению. Думаю, что такой вариант с безконечно вложенными частицами возможен, просто с каждым новым уровнем так сказать, скорости всё выше и выше, тогда и никаких проблем с задержками не будет.

Твоё объяснение меня заинтересовало. Вопрос в том, откуда то, о чём ты пишешь.
Аноним 27/02/19 Срд 13:21:36 464973102
>>464970
Бля, лучше уж тут сри, не мешай тем рыбятам, там интересная тема, причём без стремлений нарушить ваши догмы, я лишь некие мысли выложил, не более.

Какой же ты додик.
Аноним 27/02/19 Срд 13:23:46 464974103
>>464973
>рыбятам
>додик
Деревня, деревня never change.
И нищий к тому же, хехе.
Аноним 27/02/19 Срд 13:38:16 464977104
>>464972
>Так и с жидкостью и с полями всё та же проблема, откуда оно взялось вообще и из чего состоит.
первоначально была единая протяженная материя, под действие вложенного движения она мгновенно разделилась на части. поскольку пустоты не существует, то частицы не могут покинуть первоначальный объём, но они обладают движением. единственный выход для них это наяривать по кругу, образуя солнечные системы. наяривая по кругу они стачиваются до идеальных шариков с осевым вращением, а осколки без осевого вращения заполняют промежутки, бесконечно делятся и объединяются, образуя идеальную жидкость.
спин, осевое вращение, защищает частицу от объединения с другими.
>Думаю, что такой вариант с безконечно вложенными частицами возможен, просто с каждым новым уровнем так сказать, скорости всё выше и выше, тогда и никаких проблем с задержками не будет.
думаю что нет, потому что само признание бесконечности подразумевает, что где-то там в бесконечности существует пустота, которая невозможна.
>Твоё объяснение меня заинтересовало. Вопрос в том, откуда то, о чём ты пишешь.
декарт, "первоначала философии", "мир или трактат о свете"
только у первого писания перевода на русский нет, забанили за правду. но благо немцы молодцы - http://www.zeno.org/Philosophie/M/Descartes,+Ren%C3%A9/Prinzipien+der+Philosophie/3.+Von+der+sichtbaren+Welt

Аноним 27/02/19 Срд 13:38:47 464978105
7lbydds5b4t11.jpg (64Кб, 720x481)
720x481
Аноним 27/02/19 Срд 13:43:49 464980106
>>464977
>пустота, которая невозможна
Аристотель, ты? охуевший дурачок тот еще был, кстати
Аноним 27/02/19 Срд 13:46:40 464981107
>>464977

Интересная идея, почитаю. Сам сторонник того, что всё таки из небытия так сказать появилось, так что первопричина есть точно.
Аноним 27/02/19 Срд 13:50:55 464982108
>>464980
>Аристотель, ты?
размерность это свойство, а свойством может обладать только материальное тело. "ничто" не может обладать свойством, потому что его нет, оно не существует.
пустота родилась из-за невежества людского, раз не вижу, то нет. а как можно увидеть носитель света? как можно увидеть среду, через которые распространяются импульсы, которые мы зовем светом, где конечная частица в цепочке выступает в роли фотона. как можно увидеть частицы, образующие идеальную жидкость и заполняющие промежутки между сферами? тупые человечки.
но вообще под пустотой подразумевается вакуум, то есть отсутствие молекул. эм волны то продолжают распространяться, а значит есть среда для волн.
Аноним 27/02/19 Срд 13:54:09 464983109
>>464982
>эта мелкобуквенная шизофазия
Весна идет, весне дорогу :3
Аноним 27/02/19 Срд 13:59:29 464988110
>>464981
>Интересная идея, почитаю.
у декарта хорошая модель эфира, рабочая. но всё же на некоторых этапах следует делать поправку на 17 век. не все процессы он верно понимает, отсюда не всё верно объясняет. электричество тогда было вообще на уровне статики.
но главное иметь саму модель эфира - это основа. а понять процессы не составит труда.
вот например трехгранные спиралевидные частицы декарта очень похожи на нейтрино и тд.
Аноним 27/02/19 Срд 14:00:46 464989111
>>464983
>я нихуя не понимаю, и ответит мне нечем, но я мамкну, что я тут
давай, давай, пиздуй
Аноним 27/02/19 Срд 14:02:38 464990112
изображение.png (287Кб, 360x540)
360x540
>>464989
>я мамкну
Кто о чем - а школьник о мамках.
Аноним 27/02/19 Срд 14:06:11 464991113
151160451313575[...].jpg (357Кб, 1440x1800)
1440x1800
>>464990
>школьник
действительно, школьнику везде школьники мерещатся
Аноним 27/02/19 Срд 14:09:10 464992114
>>464991
Зарепортил аватарку.
Аноним 27/02/19 Срд 14:10:22 464993115
Аноним 27/02/19 Срд 14:12:09 464994116
>>464993
>этот обиженный школошизик
Ясно.
Аноним 27/02/19 Срд 14:15:49 464995117
>>464994
>я невнимашка и школоеб, я нихуя не могу вам ответить на научном языке, но я буду вас репортить и играть в уебищные подростковые подкольчики пикчами - агагага как смешно, я вас подколол, агагага
малыш, ты тредом ошибся, пиздуй отсюда, я тебе уже третий раз повторяю. есть что по теме - ты нам пиши, а пока разговор окончен.
Аноним 27/02/19 Срд 14:20:25 464996118
>>464995
>КУДАХ БАБАХ КУКАРЕКУ
Этот школодебил совсем порвался, замените.
>нам
Оно еще и шизик, хаха.
Аноним 27/02/19 Срд 15:27:27 465011119
По-хардкору поясните за исследования методов поиска внеземных цивилизаций на данный момент.
Аноним 27/02/19 Срд 15:39:20 465013120
Аноним 27/02/19 Срд 15:57:23 465015121
Были ли в истории науки эксперименты по аберрации света от взаимонеподвижных источника и приемника находящихся на поверхности Земли? Накидайте ссылок на эксперименты если есть.
Аноним 27/02/19 Срд 16:07:19 465017122
Аноним 27/02/19 Срд 16:30:37 465028123
Аноним 27/02/19 Срд 16:43:06 465031124
>>465011
На снимках Марса не нашли макдональдсов и нефтянык вышек, а на Землю никто не прилетел с фанфарами и пасом => люди одни во вселенной.
Разве можно спорить с такой логиков?
Аноним 27/02/19 Срд 16:54:27 465036125
Аноним 27/02/19 Срд 16:54:31 465037126
>>465015
Какая может быть аберрация если приемник неподвижен относительно источника, за счет чего она возникает?

Ну был такой эксперимент Майкельсона, например, он показал, что движение Земли никак не влияет на движение лучей света в системе отсчета, связанной с Землей — вне зависимости от положения интерферометра относительно направления движения Земли фотоны по нему летели одинаково.
Аноним 27/02/19 Срд 17:00:08 465038127
>>465011
>По-хардкору поясните за исследования методов поиска внеземных цивилизаций на данный момент.
Есть маняуравнение Дрейка, где половина аргументов взяты с потолка.
Есть уверенность, что инопланетяне пытаются связаться с нами непременно наиболее привычным нам способом: взять огромную антенну и ёбнуть в сторону потенциально обитаемой звезды Hello World на УКВ.

Кароч, сидят папуасы на острове и пускают дымы в небо, потому что по слухам внук вождя нахуячившись айяхуаски однажды увидел в небе железную птицу, а шаман придумал, что небесные люди живут на облаках и, если им подать сигнал, привезут нямку.
Аноним 27/02/19 Срд 17:02:08 465039128
>>465036
Нет. При чём тут фуррфагство вообще? Это /спц/, а не /фур/
27/02/19 Срд 17:05:41 465040129
>>465039
И правда не ты а жаль
Аноним 27/02/19 Срд 17:08:39 465041130
>>465037
>Какая может быть аберрация если приемник неподвижен относительно источника, за счет чего она возникает?
Если фотон движется как волна в среде, то он может сноситься этой средой, если она движется относительно поверхности Земли.

У Майкельсона интерферометр был, а он типа чувствителен на разность фаз, получающуюся от прохождения светом разных путей - это как бы сложнее чем просто померить отклонение луча света. А значит логичнее ловить сначала аберрацию, чем интерференцию. Какие опыты по этой теме проводились и с каким результатом?
Аноним 27/02/19 Срд 17:11:26 465042131
>>465041
>Если фотон движется как волна в среде, то он может сноситься этой средой, если она движется относительно поверхности Земли.
Да и частица может тоже сносится средой, только меньше.
Аноним 27/02/19 Срд 17:54:42 465046132
>>465041
Крошечные изменения скорости мерять гораздо проще, чем крошечные отклонения направления, поэтому и использовались интерферометры. В любом случае, никакого сноса не было обнаружено, скорость света во всех направлениях одинакова.

В 21 веке даже троллить эфиром и тупостью в целом — дурной вкус.
Аноним 27/02/19 Срд 18:09:11 465047133
>>465046
В интерферометре разность фаз может компенсироваться за счет сокращения тел, а аберрация никак не скомпенсирована, вроде.
Аноним 27/02/19 Срд 18:35:47 465049134
>>465047
>В интерферометре разность фаз может компенсироваться за счет сокращения тел, а аберрация никак не скомпенсирована, вроде.

Каким образом, какие именно тела сокращаются, у тебя шизофазия, что ли?

Сокращение тел пропорционально √(1-v²/c²), а разность фаз в эфирном манямирке должна быть пропорциональна v/((c+v)(c-v)), т.к. сначала свет движется вперед со скоростью c+v за время 1/(c+v), а потом назад со скоростью c-v за время 1/(c-v), складываем и получаем 2v/((c+v)(c-v)).

Тут не надо быть семи пядей во лбу, чтобы понять, что это разные формулы и при повороте интерферометра вокруг своей оси (и соответственному изменению проекции v на направление движения) они будут изменяться по разному закону и тут же перестанут компенсировать друг друга.

Еще один такой пост и я не буду тебя кормить до следующего переката, придумывай что-то получше.
Аноним 27/02/19 Срд 19:18:50 465061135
изображение.png (1646Кб, 1280x720)
1280x720
Слушайте.
А у нас, у людей, хоть одна фотка звезды какой-нибудь кроме солнца есть?
Не просто точка, а хоть что-нибудь.
Или не существует телескопов с таким зумом, чтоб хоть что-нибудь сфоткать?
Аноним 27/02/19 Срд 19:29:00 465063136
image.png (214Кб, 580x442)
580x442
image.png (188Кб, 467x640)
467x640
image.png (191Кб, 640x415)
640x415
image.png (137Кб, 640x360)
640x360
Действительно были такие наработки? Или это просто чьи-то фантазии?
Аноним 27/02/19 Срд 19:50:26 465066137
>>465063
В самом деле НАСАвский концепт, но не более.
Аноним 27/02/19 Срд 19:52:37 465067138
Аноним 27/02/19 Срд 20:00:09 465068139
>>465049
>этот залетный деревенский шизик из обо/сцая - >>465047
>у тебя шизофазия, что ли?
Похоже ты начинаешь о чем-то догадываться, хаха.
Аноним 27/02/19 Срд 20:05:30 465069140
>>465046
>В любом случае, никакого сноса не было обнаружено, скорость света во всех направлениях одинакова.
это потому что замеры производились в плотной среде, то есть в нижних слоях атмосферы. тут воздух препятствует свободному движению плотноприжатых сфер. замеры нужно производить в вакууме, на значительном удалении от поверхности земли.
Аноним 27/02/19 Срд 20:23:09 465072141
>>465046
>Проводить эксперименты даже на ГСО
>Что-то утверждать

Вспомнились сенсации про странное ускорение вояджеров.
Аноним 27/02/19 Срд 20:26:13 465073142
>>465069
Уроки-то хоть сделал?
Аноним 27/02/19 Срд 20:50:59 465080143
>>465049
>разность фаз в эфирном манямирке должна быть пропорциональна v/((c+v)(c-v))
Ты тут, как минимум, с размерностями напутал. И теории в добавок не знаешь.
Аноним 27/02/19 Срд 20:54:12 465082144
>>465049
Лоренц как раз объяснял неудачу Майкельсона реальным сокращением длин в направлении движения ветра.
Аноним 27/02/19 Срд 21:02:12 465083145
калейдоскоп.jpg (161Кб, 620x465)
620x465
звездное небо.jpg (190Кб, 800x800)
800x800
я знаю ещё одну фишку, каждая солнечная система состоит из вихря плотноприжатых сфер и на границе этих вихрей свет преломляется. то есть мы получаем на небосводе обыкновенный калейдоскоп, в котором каждая звезда или галактика переотражается сотни раз, под разными углами и с разной светимостью.
то есть в реальности большая часть звезд это фейк, мираж, их нет на самом деле. и тем более прикольно наблюдать как астрономы дрочат на эти миражи, изучают их. хотя какая-то часть реально существует, но это малая доля.
Аноним 27/02/19 Срд 21:06:40 465084146
>>465082
>в направлении движения ветра
ебать, да нет никакого ветра, эфир у поверхности земли существует в виде замкнутых колечек, у него скорости валом, а молекулы мешают движению.
чтобы смещение измерить нужно в космосе замеры делать, но даунам это не понять. это же секта отрицателей эфира, которые дрочат на ебштейна и веруют в мифические поля, что сродни полтергейстам или духам, ведьмам.
Аноним 27/02/19 Срд 21:23:22 465088147
>>465083
>>465084
Ацюковский - типичный пример старческой деменции, маразм в медицинском смысле.
Ну а ты такой с рождения.
Аноним 27/02/19 Срд 21:26:18 465089148
>>465084
>нет никакого ветра у поверхности
Миллер, Маринов и другие товарищи с тобой не согласились бы.
>нужно в космосе замеры делать
Да, было бы не плохо, там влияния атмосферы нету, скорость ветра должна быть не хилая, по идее.
Аноним 27/02/19 Срд 21:29:43 465090149
>>465088
>Ну а ты такой с рождения.
гений, великий реформатор и страх всех ебштейновских сект, да?
ну а ты у нас кто? учебникодрочер? что малышу дали в ротик, то он и сосет? типичный быдлопотребитель без пытливого ума и страсти к познаниям - ясно)
Аноним 27/02/19 Срд 21:33:07 465091150
Надеюсь, мочератор скоро очнется и снесет весь тред, кроме оп-поста.
Аноним 27/02/19 Срд 22:21:57 465112151
достаточно снести оффтоп школодрона:
>Уроки-то хоть сделал?
>Этот школодебил совсем порвался
>Миша, ты?
ну и в том же духе...
Аноним 27/02/19 Срд 22:42:14 465119152
>>465090
>учебнико
>сосет
>быдлопотребитель
Школьник as is, типичнейший и обычнейший.
>>465091
>кроме оп-поста.
Wrong. Нахуй этот тред вообще, отстойник для дебилов в спейсаче не нужен уже и так есть треды машкодроча и покпосмос
Аноним 27/02/19 Срд 22:42:38 465120153
>>465112
Оно обиделась, хаха.
Аноним 28/02/19 Чтв 00:07:36 465156154
1551297136521[1].jpg (121Кб, 1566x1196)
1566x1196
Объясните, что такого в этом твитте, что его на 4чане репостят?
Аноним 28/02/19 Чтв 00:15:18 465157155
Это, космическая радиация.
1)Нахуй она нужна?
2) На скольно она мешает бороздить космос?
Аноним 28/02/19 Чтв 00:29:33 465162156
>>465157
>Нахуй она нужна?
Хороший вопрос.
Задай его себе, девочка.
Аноним 28/02/19 Чтв 00:31:33 465163157
>>465162
Но я ппишёл в тред тупых вопрососв, отвечай.
Аноним 28/02/19 Чтв 00:36:15 465166158
>>464812
Гугли "тонкая настройка вселенной". Это не ответ на твой вопрос, но дискутивное направление научной мысли +/- параллельное твоему замешательству. Все это ебола, конечно, скорее из области научной философии, нежели научного подхода, но раз тебе интересно...
Аноним 28/02/19 Чтв 00:39:12 465167159
>>464821
Нихуя ты умный. Ну давай, расскажи мне за фундаментальную разницу между гравитационной постоянной и отношением диаметра окружности к ее длине, особенно с учетом того, что число пи применяется в физических формулах направо и налево, подразумевая собой ровно такую же, как и те, что ты называешь физическими константами, характеристику мироустройства.
Аноним 28/02/19 Чтв 00:47:43 465172160
>>465163
>я ппишёл
Ути какая няка :3
Аноним 28/02/19 Чтв 00:50:36 465173161
>>465167
>отношением диаметра окружности
Эйлеровскую приколюху в твоей шкалке еще не проходили значит.
Ясно.
Понятно.
Аноним 28/02/19 Чтв 01:06:10 465179162
>>465167
Отношение диаметра окружности к длине в евклидовом пространстве справедливо абсолютно в любой Вселенной. Оно никак не зависит от параметров физического мира, а только от аксиом геометрии. Кроме того, это отношение для реальной окружности отличается от пи, хотя и на какие-то ебически неуловимые доли, потому что Вселенная искривлена и нихуя не евклидова. Оно применяется в физических формулах, вот только в физике налево и направо пренебрегают мелкими погрешностями.

А вот гравитационная постоянная ниоткуда не следует. Ну или, по крайней мере, мы не знаем, откуда.
Аноним 28/02/19 Чтв 01:37:35 465182163
>>465179
>от аксиом геометрии
Еще один.
Аноним 28/02/19 Чтв 11:42:43 465203164
>>465119
Если бы ТТВ не было, половина этих самых тупых вопросов появилась бы отдельными тредами.
Аноним 28/02/19 Чтв 12:12:35 465210165
>>465203
>половина этих самых тупых вопросов появилась бы отдельными тредами.
Банить к хуям, как минимум на месяц.
Быдлодебилы не нужны вообще, очевидно на примере этого треда.
Аноним 28/02/19 Чтв 12:16:04 465211166
>>465210
Вахтёр, плиз. Если и банить, то таких, как ты
Аноним 28/02/19 Чтв 12:27:24 465213167
15484156567600.png (474Кб, 748x636)
748x636
Аноним 28/02/19 Чтв 13:36:55 465226168
Почему все кратеры круглые? Они такими могут быть быть только при прямом вертикальном падении. А они все всегда падают под углом, значит, и кратеры должны быть яйцеобразной формы с ударной воронкой ближе к узкой части.
Аноним 28/02/19 Чтв 13:38:55 465227169
>>465226
>Они такими могут быть быть только при прямом вертикальном падении
Дебил, блять.
Аноним 28/02/19 Чтв 14:18:43 465230170
>>465211
Ну можно еще ниграми завайпать.
Лично тебе что ближе?
Аноним 28/02/19 Чтв 14:22:01 465231171
>>465226
Потому что скорость удара настолько высока что происходит взрыв
Аноним 28/02/19 Чтв 14:23:10 465232172
Аноним 28/02/19 Чтв 15:05:37 465234173
Аноним 28/02/19 Чтв 15:39:02 465236174
Почему звёзды и планеты круглые если должны быть дискообразные?
Аноним 28/02/19 Чтв 16:01:04 465237175
himki.webm (342Кб, 480x360, 00:00:05)
480x360
>>465236
пацаны, это представитель общества плоской земли! не отвечайте дауну!
Аноним 28/02/19 Чтв 16:22:18 465239176
глаз-урагана.jpg (190Кб, 502x314)
502x314
>>465236
>Почему звёзды и планеты круглые если должны быть дискообразные?

потому что солнечная система представляет из себя вихрь из плотноприжатых сферообразных частиц эфира. поскольку, как я писал выше - пустота невозможна, то все промежутки между сферами заполняет идеальная жидкость. сферы отбрасываются наружу, а излишки идеальной жидкости собираются в центре вихря в виде крупной капли, которую мы называем солнце.
звезды и наше солнце шарообразные, потому что шар это самая энергетически выгодная форма.
но это не всё. вихрь из сферообразных частиц работает как гигантский насос и отбрасывает идеальную жидкость наружу, откуда она благополучно засасывается на полюсах других солнечных систем. и в наше солнце на полюсах засасывается идеальная жидкость. там она приобретает единую скорость и поэтому объединяется в общий столб, а поскольку она проходит через ось, образованную сферообразными частицами, то столб приобретает грани и закручивается по оси. так рождаются нейтрино и прочие винтообразные частицы со спином 1/2. сфера может оставаться на месте, а винтообразная частица всегда должна лететь вперед.
когда винтообразные частицы входят в солнце, то они отламываются на короткие частички, сталкиваются в центре солнца и разлетаются к экватору. крупные бракованные частицы не могут пройти в промежутки между сферами и образуют темные пятна. Если вихрь крупный, то темные пятна разрушаются, а если мелкий, то копятся на поверхности пока полностью не застилают звезду. так образуются планеты. поэтому мы не можем увидеть маленькие звёзды, они все спрятаны. признаком, что внутри планеты находится звезда, является наличие общепланетарного магнитного поля. поскольку магнитное поле порождается винтообразными частицами, которые вылетают из звезды. поэтому планеты тоже круглые. чем меньше звезда, тем более тяжелые атомы образуются при дроблении темных пятен.
Аноним 28/02/19 Чтв 16:23:04 465240177
Аноним 28/02/19 Чтв 16:25:56 465241178
Аноним 28/02/19 Чтв 16:36:41 465242179
>>465239
>>465240
>>465241
Два дебила - это сила.

И они еще говорят что ТТВ нужен.
Аноним 28/02/19 Чтв 18:11:07 465253180
>>465237
Просто по логике центробежная сила должна размазывать тела в блинчик, а этого не происходит, чому?
Аноним 28/02/19 Чтв 18:28:40 465254181
image.png (132Кб, 600x499)
600x499
>>465253
А балерину почему центробежная сила не размазывает в блинчик?
Аноним 28/02/19 Чтв 19:51:52 465266182
>>465254
>А балерину почему центробежная сила не размазывает в блинчик?
Ну это смотря до каких скоростей её раскрутить)
Аноним 28/02/19 Чтв 20:00:49 465271183
>>465254
А ничего что юбка стала блинчиком?
Аноним 28/02/19 Чтв 20:11:19 465275184
>>465271
>А ничего что юбка стала блинчиком?
это не юбка - это гироскоп.
Аноним 28/02/19 Чтв 20:46:35 465281185
Как вы относитесь к строительству офиса Роскосмоса за 25 миллиардов рублей?
Аноним 28/02/19 Чтв 20:49:20 465283186
>>465281
В соответствующем треде кормить перестали?
Аноним 28/02/19 Чтв 21:02:41 465285187
>>465275
это пояс астероидов как у сатурна
Аноним 28/02/19 Чтв 21:02:58 465286188
>>465266
воооот, твоя извилина начинает что-то подозревать
Аноним 01/03/19 Птн 00:27:08 465337189
Что с этим тредом не так? Почему у меня ощущение, что я в /б, даже привкус характерный появился.
Аноним 01/03/19 Птн 00:28:08 465338190
>>465281
Космическим масштабам и распил соответствующего уровня.
Аноним 01/03/19 Птн 03:32:46 465364191
Почему нельзя направить Хаббл на Луну и сфотографировать следы астронавтов?
Аноним 01/03/19 Птн 07:46:12 465373192
>>465364
Разрешения не хватит, Луна слишком далеко. Даже следы на Земле еле-еле хватило бы разрешения разглядеть, но атмосфера мешает.
Аноним 01/03/19 Птн 07:50:22 465374193
Есть ли скорость, быстрее скорости света?
Аноним 01/03/19 Птн 10:52:33 465402194
Аноним 01/03/19 Птн 11:16:50 465411195
>>465402
Читать умеешь? "Тред тупых вопросов". В ОП-посте не показан предел тупости вопросов. Так что давай-ка ты или держи свой обоссаный рот закрытым или отвечай по делу на вопросы.
Аноним 01/03/19 Птн 11:50:48 465415196
>>465402
>спросил ударник Дмитрий, скролля двачик на телефоне под партой.
Аноним 01/03/19 Птн 12:40:28 465420197
>>464509
Есть ответ на этот вопрос?
Аноним 01/03/19 Птн 13:23:11 465436198
Я тут подумал, ведь можно же создать цепь замкнутого цикла и жить на других планетах. Можно было бы даже подземные города строить.
Аноним 01/03/19 Птн 14:39:10 465438199
Если у Бурана движков не было, зачем его сбоку цепляли, как Шаттл?
Аноним 01/03/19 Птн 14:54:08 465440200
>>465438
Украли компоновку не задумываясь.
Аноним 01/03/19 Птн 16:53:55 465470201
>>465420
>While gravitationally lensed light sources are often shaped into an Einstein ring, due to the elongated shape of the lensing galaxy and the quasar being off-centre, the images form a peculiar cross-shape instead.[3]
Короче, обычные еврейские отговорки.
Аноним 01/03/19 Птн 16:57:14 465472202
>>465436
Построй сначала город на дне океана, глубоко под землей, в воздухе и на полюсах, а потом суйся осваивать другие плонеты.
Аноним 01/03/19 Птн 17:03:18 465473203
>>465436
>создать цепь замкнутого цикла и жить на других планетах
Мы на земляхе пока такое не можем провернуть, что уж говорить про другие планеты. На, почитай: https://ru.wikipedia.org/wiki/%D0%91%D0%B8%D0%BE%D1%81%D1%84%D0%B5%D1%80%D0%B0-2

Особенно вот это:
>Однако через несколько недель жизнь людей, живущих натуральным хозяйством, нарушилась. Микроорганизмы и насекомые стали размножаться в неожиданно больших количествах, вызывая непредвиденное потребление кислорода и уничтожение сельскохозяйственных культур (использование ядохимикатов не предусматривалось). Обитатели проекта стали терять в весе и задыхаться. Учёным пришлось пойти на нарушение условий эксперимента и начать поставку внутрь кислорода (23 тонны) и продуктов (эти факты скрывались и были разоблачены впоследствии). Первый эксперимент закончился неудачей: люди сильно потеряли в весе, количество кислорода снизилось до 15 % (нормальное содержание в атмосфере — 21 %).
01/03/19 Птн 19:27:25 465486204
Аноним 01/03/19 Птн 19:41:00 465492205
>>465374
Есть. Скорость сближения двух фотонов, например, летящих навстречу друг к другу, равна как минимум 2c - это очень легко экспериментально проверить.
Аноним 01/03/19 Птн 19:55:01 465501206
Аноним 01/03/19 Птн 21:30:04 465552207
Вопрос, если метан и кислород в жидком состоянии, то можно ли просто открыть вентиля и подать на камеру сгорания?


Будет ли выше кпд если камера сгорания будет обогревать жидкий метан и кислород?

И какая будет разница в кпд по цирфам если сравнивать с подачей на 150 атмосферах? 40% разницы или 400%?
Аноним 01/03/19 Птн 21:52:29 465561208
Если б космос проводил звук, какой бы он был ну кроме ты хуй/мамку ебал, само собой? Насколько громкий, например?
Аноним 01/03/19 Птн 22:08:23 465580209
>>465561
Твой вопрос взаимоисключающий. Звук зависит от среды, в которой он распространяется. В космосе звук не распространяется, потому что такой среды нет.
Это всё равно что спросить, как плавается в осушенном бассейне. Никак блядь.
Аноним 01/03/19 Птн 22:34:25 465605210
>>465580
Ну, во-первых, среда там есть и звук тоже. Но меня всё-таки интересует обычный воздух, но чтоб не гравитировал никуда.
https://www.youtube.com/watch?v=f_a9QqzCF14
Аноним 01/03/19 Птн 23:40:38 465667211
Аноним 02/03/19 Суб 01:12:43 465687212
>>465552
Слесаря отроют вентиля, вспыхнут огня и наши корабля выйдут на орбита.
Аноним 02/03/19 Суб 02:18:29 465692213
1+.png (33Кб, 845x732)
845x732
Поясни за мою идею.
Когда мы в космосе проблема утечки состоит в том, что при нарушении герметизации из-за перепада температур воздух утекает.
В сайфае-космоопере "Звёздные Войны" эта проблема решается некими полями, позволяющими проникать кораблям сквозь этот барьер, но не дающая утечь воздуху.
И я подумал, а что если дать утечь воздуху? Ведь воздух будет капельками прилепать к стенкам корабля, особенно если это узкие стенки в туннеле шахты. В космосе и воздух будет иметь определённый уровень поверхностного натяжения (как мне кажется). То есть, имея трубу определённой длины - весьма большой, как я полагаю - можно создать один небольшой открытый шлюз, позволяющий держать открытым гипотетическую станцию для прямого доступа в открытый космос, не боясь при этом растерять драгоценный воздух и не используясь при этом сайфай допущениями волшебный силовых полей, а ограничиться реальной физикой движения потоковых масс и частиц. Что скажешь, анон? Подозреваю, идея моя легко допустима, если будет туннель длинной в десяток другой километров, заворачиваемый так и сяк, и имеющий какую-то определённую поверхность внутренней стороны, соблаговаляющую удержанию воздуха.
Аноним 02/03/19 Суб 02:37:13 465693214
>>465692
>туннель длинной в десяток другой километров
Ну и какой смысл в такой херне, если можно просто шлюзы поставить на всём протяжении и укоротить в 100 или 1000 раз? Алсо, воздух будет утекать из этой штуки просто потому что давление будет стремиться выровняться между всеми условно говоря "сосудами", может это будет происходить медленно, не быстро, но это и не должно происходить быстро, просто банально из-за длинны этой ебаторики на несколько километров и объемов газа в ней, я боюсь представить какого размера этот шарик с 1атм в нижней части картинки.
Аноним 02/03/19 Суб 03:14:07 465697215
1+.png (23Кб, 845x732)
845x732
>>465693
А если так и за счёт вращения отталкивать воздух?
Ну и хуй с ним - щепотку силовых полей как в ЗВ (только я х/з, что это за поля должны быть)
> Ну и какой смысл в такой херне
Чтобы при каждом открытии-закрытии шлюза драгоценный воздух не выбрасывало под силой собственного давления. При многократном использовании даже утечка пары млЛ воздуха в долгосрочной перспективе приведёт к иссяканию воздуха внутри до непригодных для жизни величин
> я боюсь представить какого размера этот шарик с 1атм в нижней части картинки.
Форма сперматозоида - условность. Если проще представить, возьмём некую маленькую Звезду Смерти (не по функционалу, а по размерам) и сделаем так, чтобы один туннель наматывал круги по поверхности превращая всю поверхность в огромный лаз со входом из космоса например на внешней стороне северного полюса, и безшлюзным выходом во внутреннее убранство станции на внутренней стороне южного полюса.
Аноним 02/03/19 Суб 04:59:15 465712216
>>465561
Всё бы шумело и пердело отовсюду. Представь, какой хотя бы от звёзд шум бы издавался. Вряд ли в таких условиях у организмов бы вообще слух развился. Я думаю, было бы что-то вроде того же белого шума
Аноним 02/03/19 Суб 08:02:45 465724217
>>465697
Создашь гравитацию за счет вращения и выйдет то же самое, что и у нас на Земле — даже с 1g атмосфера тянется на сотни километров вверх и все равно по чуть-чуть утекает наружу в космос.

Чтобы утечку снизить до уровня погрешности, тоннель должен быть сотни километров длиной, при том, что можно построить пару шлюзов длиной в несколько метров и добиться того же самого эффекта.

>Чтобы при каждом открытии-закрытии шлюза драгоценный воздух не выбрасывало под силой собственного давления

Какого давления? У нормальных людей шлюз герметично изолирован как от космоса, так и от основного объема станции, и после закрытия дверей воздух из него откачивают.
Аноним 02/03/19 Суб 09:45:57 465744218
>>464488 (OP)
Солнце погасло нахуй.
Ну а если серьезно, там солнечная активность существенно просела. Что это значит для Земли? Какие могут быть последствия? Как часто такое происходит? Когда ожидается восстановление активности?
Аноним 02/03/19 Суб 09:52:42 465746219
Аноним 02/03/19 Суб 11:09:55 465818220
Будут ли запускать для луны/марса аналог жпс при колонизации? Есть ли уже какие-то проекты?
Аноним 02/03/19 Суб 11:11:15 465821221
Аноним 02/03/19 Суб 11:36:56 465853222
image.png (443Кб, 640x911)
640x911
image.png (245Кб, 611x852)
611x852
image.png (275Кб, 625x852)
625x852
image.png (319Кб, 620x863)
620x863
Аноним 02/03/19 Суб 11:39:29 465857223
>>465853
Спс. Я так понимаю, там нужно кучу человек поселить, чтобы развертывание было оправданным.
Аноним 02/03/19 Суб 11:56:40 465869224
>>465724
> и после закрытия дверей воздух из него откачивают.
Как? Там же уже вакуум. Не будешь же ты мне втирать про вакуумные вентиляторы, которые дуют эфиром на водяную пыль.
Аноним 02/03/19 Суб 12:42:59 465922225
Mira1997.jpg (9Кб, 350x350)
350x350
240px-Betelgeus[...].jpg (8Кб, 240x240)
240x240
240px-Thesurfac[...].jpg (8Кб, 240x240)
240x240
240px-VLTIrecon[...].jpg (8Кб, 240x240)
240x240
Аноним 02/03/19 Суб 12:49:40 465930226
>>465869
Где вакуум? Когда шлюз закрывают с внутренней стороны, в нем одна атмосфера, как и в самой станции, которую потом откачивают обычными вакуумными насосами.

До <0,01 Па несколько кубометров можно откачать за вполне разумное время.
Аноним 02/03/19 Суб 13:02:59 465943227
videoplayback.webm (4451Кб, 640x360, 00:00:58)
640x360
Аноним 02/03/19 Суб 13:06:45 465949228
>>465930
Нет, подожди.
Значит, капли воздуха дрыгаются в вакууме космоса. Есть вакуумный насос. Какие образом частицы будут в вакуумный насос идти по тяге, если нет разницы давления между вакуумом пылесоса и вакуумом космоса?
Аноним 02/03/19 Суб 13:32:35 465973229
>>465818
Конечно будут. По сравнению со стоимостью колонизации (которая еще хуй знает когда будет), накидать пару десятков автономных спутников - считай что бесплатно.
Аноним 02/03/19 Суб 15:39:38 466058230
15513614264520.png (1567Кб, 1430x645)
1430x645
Что на пике?
Аноним 02/03/19 Суб 15:51:38 466063231
Аноним 02/03/19 Суб 15:53:57 466065232
>>466058
Макет спускаемого аппарата Венеры-4.
Аноним 02/03/19 Суб 15:54:58 466066233
>>466065
>Венеры-4
Спасибо...
Аноним 02/03/19 Суб 15:55:43 466067234
>>466058
А есть ли побольше фоточек этой штуки с разных ракурсов?
Аноним 02/03/19 Суб 19:08:13 466171235
>>465492
>Einstein edition
>этот пост
Аноним 02/03/19 Суб 19:11:25 466174236
>>465746
>Ничего, никакие, каждые 11 лет, в следующем году.
а солнечная активность имеет периоды в 11 лет потому что идеальная жидкость засасывается на полюсах солнца и выбрасывается по экватору через промежутки между плотноприжатыми сферы эфира. но выбрасывается уже в виде спиралевидных частиц с гранями, бракованные частицы не проходят в промежутки между сферами и остаются на поверхности солнца, формируя темные пятна.
когда поток идеальной жидкости в полюса большой, то и пятен много, а когда поток слабый, то пятен мало.
Аноним 02/03/19 Суб 19:18:55 466185237
Я не нацсрача (да и вообще другого любого срача) ради, а просто так аккуратненько, корректненько так, мягонько и ненавязчиво так, никого ни в чем не возвышая и не принижая так спрашиваю - был ли полет Бурана чем-то охуенным и выдающимся с технической точки зрения?
Мог ли Спейс Шаттл садиться на автопилоте?
Аноним 02/03/19 Суб 19:27:51 466188238
>>466185
Хуета полная. Спиженный с Шатла планер и почти вся начинка. Что тут выдающегося? Автопосадка? Охуеть новинка. К моменту полета Бурана самолеты уже лет 30 умели сами садиться.
Буран - это тупое говно тупого говна. Почему думаешь на него забили после единственного полета? Потому что это тупое говно тупого говна.
Аноним 02/03/19 Суб 19:55:31 466211239
>>466185
>был ли полет Бурана чем-то охуенным и выдающимся с технической точки зрения?
Нет, не был.

>Мог ли Спейс Шаттл садиться на автопилоте?
После небольших доработок мог, но сделано этих доработок не было, в основном потому, что попросту не было запроса от НАСА.

Если кто-то говорит, что не мог, то отвечай, что ну тогда Буран не мог ни выводить грузы, ни возить людей, т.к. для этого требовались доработки гораздо большего масштаба.
Аноним 02/03/19 Суб 19:59:10 466214240
>>466211
>Буран не мог ни выводить грузы
Но он вывел.
Аноним 02/03/19 Суб 20:00:24 466215241
>>466067
Помогите пожалуйста.
02/03/19 Суб 20:02:31 466217242
>>466215
Google -> Венера-4 Мемориальный музей космонавтики
Аноним 02/03/19 Суб 20:03:42 466220243
>>466185
В шаттле по идеологическим причинам было сделано так, чтобы некоторые решения нельзя было принять без человека. Например, выпуск шасси. Так-то он обычно всё равно садился на автоматике. Так что да, шаттл мог сесть автономно, но только на брюхо. Это такие свои загоны были у руководства наса в то время.
Аноним 02/03/19 Суб 20:09:17 466227244
>>466220
А в чем смысл, чтобы совки каким-то образом не спиздили управление чтобы посадить шатол к себе?
Аноним 02/03/19 Суб 20:12:51 466233245
>>466227
Чтобы у астронавтов не было ощущения, что за них партия решает, когда выпускать шасси.
Аноним 02/03/19 Суб 20:34:01 466249246
>>466227
На ручном управлении были только четыре так называемых "необратимых" действия, чтобы астронавты не бугуртили: выпуск шасси (которые не имели приводов и тупо выпадали через щит, обратно не задвигались), выдвижение приемника воздушного давления (обратно не задвигался), выпуск тормозного парашюта (аналогично) и включение вспомогательных силовых установок.

Срабатывание любого из них раньше времени (кроме разве что ВСУ, где можно было очухаться до того, как они выработают весь гидразин) фактически гарантировало при посадке катастрофу а-ля Колумбия, поэтому сами астронавты были против их автоматизации.
Аноним 02/03/19 Суб 20:41:52 466254247
>>466214
Ну и что он вывел?
Аноним 02/03/19 Суб 20:49:58 466257248
bdp2.jpg (124Кб, 565x522)
565x522
bdp.jpg (136Кб, 561x518)
561x518
37kb3707.jpg (86Кб, 574x394)
574x394
Аноним 02/03/19 Суб 22:12:53 466292249
>>466211
>После небольших доработок мог, но сделано этих доработок не было
Было. После катастрофы "Колумбии".

Добавили специальный кабель, что-то там с чем-то соединявший, чтобы с земли можно было выдать команды, перечисленные в >>466249

Сделали чтобы в случае чего оставить астронавтов куковать на МКС, а корабль попытаться посадить на автомате.
Аноним 02/03/19 Суб 22:21:15 466293250
А что это за трэд?
Аноним 03/03/19 Вск 01:07:09 466342251
Если б возле сталкивающихся чёрных дур крутилась насквозь промороженная планета, то гравитационные волны мгновенно бы её равномерно прожарили, а то и испарили? Или они слабо передают энергию?
Аноним 03/03/19 Вск 09:06:18 466386252
Тёмная энергия действительно имеет массу со всеми атрибутами (инертность/собственное притяжение/всякие ото-эффекты) или это просто не имеющий (пока что?) физсмысла эквивалент из е=мц2?
Аноним 03/03/19 Вск 09:11:26 466387253
Ну и тогда уж, может ли тёмная материя, например, не иметь инерции, а только притягивать? Рассматриваются такие варианты?
>>466386
Аноним 03/03/19 Вск 10:59:35 466399254
>>465236
Потому что гравитация.
Аноним 03/03/19 Вск 11:03:55 466400255
>>466399
Чет я подумал, а почему всё вращается?
Аноним 03/03/19 Вск 12:43:30 466447256
Можно ли раскачать станцию изнутри, чтобы корабль не смог пристыковаться?
Аноним 03/03/19 Вск 13:15:15 466482257
>>466447
Дмитрий Олегович, вы законы сохранения импульса и момента импульса в школе проходили?
Аноним 03/03/19 Вск 14:38:49 466642258
>>465226
Неприятный вопрос, потому что это таки взрывы и не факт, что от метеоров.

Как пример против метеоров падение оного под Хабаровском.
Аноним 03/03/19 Вск 15:03:21 466677259
>>465231
Это по сути застывшая волна. Энергии выделяется столько, что твёрдое вещество начинает вести себя как жидкость.
Аноним 03/03/19 Вск 15:05:30 466680260
Почему у НАСА не было автоматической стыковки (до событий последних часов), хотя Совок уже лет 50 умеет в автоматическую стыковку? Казалось бы, нация с самыми мощными компьютерами мира, а в автоматическую стыковку не могут.
Тоже боятся за "слишком самостоятельные" технологии, как описано выше про Шаттл?
Аноним 03/03/19 Вск 15:22:28 466693261
Аноним 03/03/19 Вск 15:45:35 466730262
>>466693
А не словесного, а эмпирического доказательства не будет что ли? Смысл в этих разглагольствованиях?
Аноним 03/03/19 Вск 15:49:50 466731263
>>466730
Там две трети ролика рассказывается об экспериментах Роскосмоса НАСА на эту тему, как физических, так и виртуальных, на симуляторе.
Аноним 03/03/19 Вск 16:57:13 466834264
>>464488 (OP)
Почему нигде нет видео земли с орбиты?
Типа как в гугл картах снимки с космоса, только видосы. Чтоб смотреть как люди ходят, самолеты летают, etc.
Аноним 03/03/19 Вск 17:54:50 466884265
>>466834
С МКС уже дохуя лет стримят видео, смотри не хочу.

В большом увеличении снимать смысла нет, спутники слишком быстро движутся и точка съемки моментально сместится, никакой полезной информации из такой съемки не извлечь и это нахуй никому не надо.

>Чтоб смотреть как люди ходят
Максимальный уровень приближения на гугл картах, где хорошо видно людей, это вообще обычно аэрофотосъемка, а не фото со спутника, со спутника качество подерьмовее.
Аноним 03/03/19 Вск 18:01:21 466892266
>>466834
>Почему нигде нет видео земли с орбиты?
есть, плохо искал. Есть целые коммерческие компании со своими собственными спутниками, которые снимают видео любой точки Земли на заказ.
Аноним 03/03/19 Вск 18:04:37 466895267
>>466834
Ты слишком веришь голливудским фильмам, где со спутников не только видео снимают, но и разговоры подслушивают. На деле спутник хуярит по орбите со скоростью ~7 километров в секунду, город уровня Москвы он пролетит буквально за пару секунд и разумеется что сфокусироваться можно только на какой-то конкретной области и только для съемки фотографий. Тут еще надо заметить, что спутник летает на полярной орбите, а Земля под ним вращается, то есть пролетев один раз над Москвой, в следующий раз он в этой области может оказаться и через неделю и через месяц, как повезет.
Аноним 03/03/19 Вск 20:21:30 466997268
>>466895
А если геостационарный спутник? Или слишком далеко?
Аноним 03/03/19 Вск 20:31:55 467003269
>>466997
Ну ёпта, конечно далеко. 35,786 километров, а спутники ДЗЗ летают в районе 500 где-то, плюс-минус пара сотен км в зависимости от конкретных целей.

На геостационаре спутники ДЗЗ тоже есть, но специфические. Некоторые погодные, например. https://en.wikipedia.org/wiki/Weather_satellite#Geostationary
Аноним 04/03/19 Пнд 08:41:02 467120270
>>466680
С одной стороны, ручную стыковку, с содействием со стороны компьютера, проще сделать.
С другой стороны, в штатах никогда не испытывали каких-то особых проблем с использованием пилотов. Это скорее наоборот, у СССР было маниакальное стремление сделать пилота ненужным.
Аноним 04/03/19 Пнд 10:02:26 467136271
Если гигантский космический великан проткнет земную мантию гигантской соломкой и выпьет всю мантию, будет ли земная кора, литосферные плиты держаться друг на друге как кирпичи в арке, или потрескается и упадет на ядро?
Аноним 04/03/19 Пнд 12:12:04 467161272
Первая ступень Флакона + Дракон на ней, можно ли такую связку использовать как мини-BFR для межконтинентальных перелётов?
Аноним 04/03/19 Пнд 12:20:03 467162273
>>467136
Тебя это ебать не должно
Аноним 04/03/19 Пнд 12:20:03 467163274
>>467003
>Ну ёпта, конечно далеко
Просто ты не в курсе возможностей ЦРУ.
Аноним 04/03/19 Пнд 12:37:33 467166275
>>467163
Ну да, у ЦРУ законы физики свои, секретные.
04/03/19 Пнд 12:45:01 467168276
>>466731
Хуле ты ему отвечаешь? Эта зелень пытается скатить тред ТВ на уровень зогача.
Аноним 04/03/19 Пнд 12:51:33 467169277
>>467168
>поднять тред ТВ на уровень зогача.
Поправил, не благодари.
Аноним 04/03/19 Пнд 12:53:59 467171278
>>467169
Уриноприемник, уймись.
Аноним 04/03/19 Пнд 12:58:25 467172279
>>467171
Ты докукарекаешься до вайпа раздела и утопления символа доски - ЖУКОВ-ГОВНОВОЗОВ
Аноним 04/03/19 Пнд 13:09:56 467175280
>>467136
Рухнут даже без всяких трещин, такую массу хуй удержишь. Да хоть ты кору замени на графен - все равно рухнет.
Аноним 04/03/19 Пнд 13:12:37 467177281
Вопросы:
1. Правда что на МКС пьют переработанную мочу? Т.е. замкнутый цикл воды.
2. Как моются на МКС?
Аноним 04/03/19 Пнд 13:27:19 467181282
>>467177
1. Пидора вопрос.
2. Да.
Аноним 04/03/19 Пнд 13:56:00 467194283
>>467136
Ну эта хуйня будет гравитационно нестабильна в любом случае.
04/03/19 Пнд 14:49:42 467201284
Аноним 04/03/19 Пнд 16:05:18 467218285
>>467172
Навозников нетрож!
Аноним 04/03/19 Пнд 16:44:00 467232286
>>467218
На коленях умоляй, как положено.
И за этого шароеба тоже - >>467201
Аноним 04/03/19 Пнд 16:56:08 467238287
>>467161
Только если в одноразовом режиме, с реюзом на межконтинентальный перелет + торможение и посадку первой ступени не хватит дельты. В самом драконе топлива маловато, он слабый помощник.
Аноним 04/03/19 Пнд 17:03:28 467240288
>>467161
Такие перелеты это 90% орбиты, не хватит.
Аноним 04/03/19 Пнд 17:42:27 467252289
>>467177
1) Только американцы. Русские же резонно считают что таким образом можно зашкварится а и пьют ювенильную воду из пыли, которая как роса оседает на солнечных панелях.
2) Обливаются потом и это смывает грязь. Кстати американцы потом и это пьют.
Аноним 04/03/19 Пнд 17:44:36 467254290
>>467177
1 такая вода используется только в технических целях и вообще это скорее отработка технологий
2 салфетками обтираются
Аноним 04/03/19 Пнд 18:06:06 467262291
Да что уж там, мы даже тут на земле пьем то, что когда-то было чьей-то мочой.
Аноним 04/03/19 Пнд 18:13:39 467263292
Аноним 05/03/19 Втр 21:16:14 467612293
largefairingcom[...].jpg (13Кб, 264x400)
264x400
Почему редко используют такие большие обтекатели и странные формы?
Аноним 05/03/19 Втр 21:30:28 467615294
Аноним 05/03/19 Втр 21:34:49 467618295
323231.png (114Кб, 680x521)
680x521
А можно ли сделать ракету на метане и кислороде под 200 атмосферами в баллонах из композитов?


КПД будет не очень, но с другой стороны можно без нагнетателя обойтись(тоже вес)
Аноним 05/03/19 Втр 22:09:36 467630296
СпецГО Космос-3М.jpg (36Кб, 800x600)
800x600
>>467612
Обтекатели, отличные от круглых в сечении, скорее вынужденная мера, когда есть базовый ГО, но полезная нагрузка в него, сука, не влезает.
Аноним 06/03/19 Срд 09:13:21 467693297
Тёмная энергия является причиной энтропии вселенной?
Аноним 06/03/19 Срд 09:24:06 467695298
>>467693
Нет Возможно ты неправильно понимаешь значение слова энтропия
Аноним 06/03/19 Срд 11:16:00 467714299
>>467695
Вселенная расширяется из-за тёмной энергии -> остывает -> тепловая (энтропийная) смерть. Или что то там с симметриями связано, законами сохранения?
Аноним 06/03/19 Срд 11:35:36 467717300
Как пар раскручивает турбины?
Аноним 06/03/19 Срд 11:43:00 467725301
Аноним 06/03/19 Срд 12:46:27 467739302
>>467714
Полна энтропия тепловая смэрт
Аноним 06/03/19 Срд 12:50:02 467741303
>>467739
>>467714

это лишь равномерное распределение тепла во вселенной, ну и темная материя как и расширение вселенной на это ни как не влияет, только чуть-чуть замедляет случайно отправил
Аноним 06/03/19 Срд 14:25:05 467745304
>>467741
Почём нынче паскод? Нужно авторизоваться, или кукисы просто не удалять? Сколько раз было "перейдайте Абу произошла грубейшая случаная ошибка"?
Аноним 06/03/19 Срд 14:26:11 467746305
>>467714
Даже если бы Вселенная была постоянного размера, энтропия бы все равно росла. В некотором роде, энтропия - показатель "хаотичности". Собрал энергию в одном месте - уменьшил хаотичность (в этом месте). Но для этого надо ее увеличить, причем более существенно, в других местах.
Аноним 06/03/19 Срд 14:36:06 467747306
>>467746
Что является причиной энтропии? В научных термина если не трудно.
Аноним 06/03/19 Срд 15:38:07 467756307
Аноним 06/03/19 Срд 16:35:31 467759308
>>467747
Выравнивание температуры
Аноним 06/03/19 Срд 16:56:47 467760309
>>467747
Ну тащемта это постулат, он ниоткуда не следует. С другой стороны, он хорошо проверен эмирически.
07/03/19 Чтв 09:15:08 467897310
Аноним 07/03/19 Чтв 11:26:57 467915311
>>467760
>>467759
Какой-то всемирный уравнитель? Какой у него принцип? Плюс-минус-пополам? Принцип наименьшего действия/движения? Что-то такое было но не могу вспомнить. Неужели это просто упругое распределение. Хмм. Кто-бы носом тыкнул.
Аноним 07/03/19 Чтв 12:17:28 467931312
>>467897
Ну напиши нехуйню, сагователь говёный.
07/03/19 Чтв 12:38:50 467934313
>>467931
А зачем? Все равно здесь никто ничего не поймет. Про законы термодинамики и их статистическое обоснование здесь, похоже, никто и не слышал. Попробуй прочитать для начала вот это:
https://studfiles.net/preview/882529/page:35/
Аноним 07/03/19 Чтв 22:42:16 468073314
Котаны, вот тепловая смерть. Все развалилось до действительно элементарных частиц (какие бы они не были), максимум энтропии, полная однородность/инвариантность etc.
Но ведь даже в таком состоянии будут присутствовать флуктуации, которые снова приведут к созданию составных частиц, атомов, etc. Короче, кто победит - концепция тепловой смерти или концепци больцмановского мозга?
Аноним 08/03/19 Птн 00:31:13 468116315
ZFpNokDwwHQ.jpg (42Кб, 604x576)
604x576
Спейсач, поясни за черные дыры - вот никакой объект обладающий какой-никакой массой, после приближения к чОрной дыре не способен от неё съебаться.
А что с радиоволнами? Они же не обладают массой.
Что если запустить в черную дыру йоба-зонд с кучей разных датчиков, и все это дело транслировать наблюдателю через йоба антенну, то сможет ли наблюдатель получать данные в онлине т.е. за время прохождения радиоволн от зонда до наблюдателя, а не через бесконечно долгое время?
Аноним 08/03/19 Птн 04:15:39 468138316
>>468073
>которые снова приведут к созданию составных частиц
Эт схуяли?
Аноним 08/03/19 Птн 04:19:36 468140317
>>468116
>после приближения к чОрной дыре не способен от неё съебаться
После погружения под горизонт событий ващет.
>Они же не обладают массой
Энергия и масса эквивалентны в таких категориях. Само понятие массы слишком комплексное, в которое лучше палкой не тыкать, дабы кефирошизика и прочих адептов Ацюковского не возбуждать.
Судя по уровню твоих вопросов, тебе для базиса не помешало бы по тематическим статьям в википедии хотя бы пробежаться.
Аноним 08/03/19 Птн 05:22:37 468141318
>>468138
Чисто статистически. Рано или поздно это случится.
Аноним 08/03/19 Птн 15:15:50 468309319
>>467747
В детстве писал программу, рисующую узор, по алгоритму рядом искался цвет, что на единицу больше и текущий ему приравнивался, для последнего первый считался следующим, то есть цикл замыкался. Пиксели изначально получали случайные цвета, что любопытно, в конченом итоге получались плоские вихри во всём этом деле.

Позже, пытаясь смоделировать так сказать вселенную, что стремится к среднему значению, подобного не наблюдалось, всё быстро схлопывалось в один цвет.

К чему это я, к тому, что энтропии нет, это ересь, есть безконечное перетекание энергий одна в другую.
Аноним 08/03/19 Птн 20:54:40 468534320
>>468141
Нет, схуяли этот процесс станет доминантным вдруг? Ну образовался больцмановский мозг. Ну обратился обратно в стандартный кисель под действием внешних условий за пару часов-дней. С хера ли вдруг по-твоему флуктуации начнут носить упорядоченный характер, превалирующий над энтропией?
Аноним 08/03/19 Птн 23:30:32 468673321
>>468309
>безконечное
Уроки-то сделал?
Аноним 08/03/19 Птн 23:37:40 468677322
>>464488 (OP)
Сколько лет осталось существовать жизни на Земле? Видел оценки от 200 млн, до 1 млрд.
Аноним 08/03/19 Птн 23:55:05 468682323
>>468677
>Сколько лет осталось существовать жизни на Земле?
Да хуй знает, какие-нибудь бактерии-экстремофилы будут существовать практически до начала превращения Солнца в красный гигант. Разве что полное испарение всей воды может их добить раньше, но думаю под землей, да в районе полюсов останутся какие "влажные" места.
>Видел оценки от 200 млн, до 1 млрд.
Ну, собственно, из-за неоднозначности того, что считать "существованием жизни", такие и разбежки. Опять же, не стоит недооценивать способность жизни приспосабливаться. 200 млн, как мне кажется, это уж больно пессимистично.
Аноним 09/03/19 Суб 00:06:44 468683324
>>468682
Ну ок, давай будем говорить о многоклеточных организмах. Вот собственно полное испарение воды и вангуют на период
>от 200 млн, до 1 млрд
а хочется конкретики.
Аноним 09/03/19 Суб 00:09:35 468685325
>>468683
Полное испарение воды только через миллиард лет.
Аноним 09/03/19 Суб 00:12:57 468686326
>>468683
Ну я имел в виду не только испарение океанов, а полное исчезновение воды из земной коры, по крайней мере из ее верхней части. Бактерии и архебактерии замечательно живут под землей, причем хуй знает, до какой глубины.

А многоклеточная жизнь наверняка вымрет с океанами. До их полного пересыхания возможно распространение современной фауны "черных курильщиков" - эти парни к горячей воде уже привыкли (да и другие могут адаптироваться). Хотя, правда, для еды им нужны бактерии в больших количествах, так что может от "курильщиков" уйти и не смогут.
Аноним 09/03/19 Суб 00:29:08 468687327
изображение.png (660Кб, 1200x675)
1200x675
Какой смысл в Ангаре А5\А7 если она выводит в 2 раза меньше чем Falcon Heavy, того же класса. Это ведь заранее проигрышный вариант, вкладывать в продолжение её разработку, если она неспособна конкурировать на рынке и заранее устаревшая...

Объясните
Аноним 09/03/19 Суб 01:01:24 468692328
>>468677
Откуда ты увидел такие оценки и чем они аргументируются?
Аноним 09/03/19 Суб 01:02:53 468694329
>>468687
Нихуя они не того же класса. А5 примерно как фалкон, А7 чуть больше, ну как полтора, но никак не три. Но в остальном ты прав, никчемная ракета, хуй знает зачем делают, позорятся только.
Аноним 09/03/19 Суб 01:34:22 468697330
>>468687
Грузоподъёмности у вариантов Ангары подходящие, не в них дело.
Аноним 09/03/19 Суб 14:23:28 468761331
x4df18864.jpg (56Кб, 600x509)
600x509
Фапают ли космонавты на мкс? Что будет, если космонавт обмалафится во сне от длительного воздержания или пернет с подливой? Или они там в подгузниках круглосуточно, что бы последствия таких конфузов по всей станции не летали?
Аноним 09/03/19 Суб 14:52:01 468768332
>>468761
>Что будет, если космонавт обмалафится во сне от длительного воздержания или пернет с подливой
Его с позором снимут с МКС ближайшим кораблем.
Аноним 09/03/19 Суб 14:54:56 468770333
>>468761
>Фапают ли космонавты на мкс?
Падалка сказал, что да, фапают, точно так же, как полярники и подводники.
Аноним 09/03/19 Суб 17:21:36 468789334
>>468770
В рот друг другу?
Аноним 09/03/19 Суб 18:09:19 468797335
>>468770
А почему им не завезут резиновую куклу или хотя бы мастурбаторы? Это же МКС, технологии, хайтек, а они там письку дергают как в каменном веке.
Аноним 09/03/19 Суб 18:53:12 468804336
А что будет если набрать первую космическую скорость если лететь не по орбите а тупо вверх от земли?
Аноним 09/03/19 Суб 19:08:49 468806337
>>468804
Ебнешься обратно через какое-то время. Чтобы улететь кхуям нужна вторая космическая.
Аноним 09/03/19 Суб 19:11:17 468807338
>>468806


А на какую высоту можно попасть?


Какие кстати высоты для 1,2,3,4,5км в сек если вертикально переть?
Аноним 09/03/19 Суб 19:12:33 468808339
>>468807
Лень считать, но на большую. Десятки тысяч километров, если так, с потолка.
Аноним 09/03/19 Суб 19:28:08 468812340
Считается, что в галактиках дохуя темной материи, которая влияет на их структуру и скорости орбитального (вокруг центра галактик) движения звезд. Но для рассчетов всяких орбит внутри Солнечной системы никто никакую темную материю в рассчет не принимает, при этом точность охуенная. Как так? В Солнечной системе нет темной материи? Или она сидит внутрях скоплений обычной материи (Солнце, Юпитер) и поэтому похуй, какая доля массы там на что приходится?
Аноним 09/03/19 Суб 19:41:14 468813341
Erdgvarp.png (12Кб, 290x222)
290x222
>>468807
>>468808
Если прикинуть при скорости 1 км/сек, пренебрегая сопротивлением воздуха и считая g = 10 м/с2, получится 50 км. g на самом деле чуть меньше (пикрелейтед), а сопротивление воздуха у Земли на такой скорости просто нахуй всё расплавит. Если взять 8 км/с, считать, что стартуем с высоты 100 км, и взять g = 8 м/с2 (то есть для высоты ~700 км), то будет 4000 км, что, естественно, гораздо меньше реального значения, потому что там уже g будет заметно меньше.
Аноним 09/03/19 Суб 19:44:57 468814342
>>468812
>В Солнечной системе нет темной материи?
Зис скорее всего по теории. В тёмную материю есть несколько кандидатов, и звезды главной последовательности кандидатами не являются.
Аноним 09/03/19 Суб 20:25:14 468819343
Screenshot20190[...].png (42Кб, 856x627)
856x627
>>468808
>Лень считать
Сейчас прикинул дифур: r'' = -cr^-2, где r - расстояние до центра Земли, а c - коэффициент, зависящий от гравитационной постоянной и массы Земли. Пикрелейтед - то, что мне выдал Вольфрам, лол.
Аноним 09/03/19 Суб 20:51:49 468825344
>>464488 (OP)
Какого хуя научпоп надрачивает на термоядерный реактор?
Мол термоядерные реакторы достаточно мощные, чтобы завести полноценный старшип на ионной тяге. Но современные реакторы деления тоже довольно мощны, основная проблема в охлаждении контура, как решат её при термояде?

Аноним 09/03/19 Суб 20:54:15 468826345
>>468797
>А почему им не завезут резиновую куклу или хотя бы мастурбаторы?
Подозреваю, что у американцев все это есть, просто не разглашается из этических соображений. А пидорахам оно и нах ненадо, там кузмичи сидят, которые думают только о том, как бы накатить поскорее
https://www.youtube.com/watch?v=28EMnhKd4p8
Аноним 09/03/19 Суб 21:01:17 468827346
>>468825

Суть в том, что имеется в виду термоядерныей ракетный двигатель, то есть топливо для реактора является рабочим телом ракетного двигателя и уносит тепло с собой. А вообще сигма-деретринитация во все поля.
Аноним 09/03/19 Суб 21:06:38 468829347
>>468534
Ну вот моя мысль о том, что энтропия в этом случае не изменится, просто в какой-то области пространства появится более стабильная сейчас конфигурация. Примерно как на первых стадиях абиогенеза в первичном супе увеличивалась концентрация нуклеотидов, соединенных в цепочки - так как одиночные нуклеотиды быстро распадались обратно.
Более глобально - не ебут ли квантовые принципы (нулевые колебания физического вакуума) термодинамику - ведь переход в ложный вакуум не меняет энтропию системы, но приводит к изменению состояния.
У менч сейчас 39.8 если что. В таком состоянии я могу только думать.
Аноним 09/03/19 Суб 21:16:41 468831348
>>468826
>https://www.youtube.com/watch?v=28EMnhKd4p8
Вот первое, что меня удяивляет, что это вообще обнародовали.
А второе, что лахта нигде не кричит, что это фейк.
Из этого напрашаивается вывод, что сие было додобрено на высшем уровне, как утверждающеее русские скрепы и позывы глубинной русской души.
Аноним 09/03/19 Суб 21:52:19 468839349
>>468831
Просто пидорах готовят к окончательному сворачиванию космической программы, когда очередной "Союз" наебнется. Чтобы не сильно бугуртили, типа а нах нам это все, лучше путинки накатить.
Аноним 09/03/19 Суб 21:58:39 468840350
>>468789
Твоей мамке за щеку.
Аноним 09/03/19 Суб 22:15:12 468843351
>>468812
Плюс еще средняя плотность темной материи по Млечному пути более чем на десять порядков ниже, чем средняя плотность Солнечной системы в пределах орбиты Нептуна.

Если темная материя не внутри массивных тел сидит, а более-менее равномерно распределена по пространству, то заметить вносимое ей ничтожное отклонение на таких масштабах фактически невозможно, надо смотреть хотя бы на масштабе нескольких световых лет.
Аноним 09/03/19 Суб 22:55:12 468851352
>>464488 (OP)
Почему на втором оппике получается КРЕСТ ЭЙНШТЕЙНА, хотя по идее должны получаться окружности либо дуги?
Аноним 09/03/19 Суб 23:53:14 468870353
Аноним 10/03/19 Вск 00:45:52 468877354
>>468827
Нерва на делении основана же
Аноним 10/03/19 Вск 02:18:02 468897355
С какой скоростью надо пролететь над Ганимедом, чтобы нагреться об его тонкую атмосферу ?
Аноним 10/03/19 Вск 10:46:37 468931356
Почему ученые не пытаются создать сингулярности в лабораторных условиях?
Аноним 10/03/19 Вск 12:28:50 468943357
>>468897
с очень большой скоростью, на такой скорости тебе даже просто отдельные частицы из межпланетного пространства страшны.
Аноним 10/03/19 Вск 12:44:55 468946358
>>468931
Нет ускорителей такой мощности, да и даже если были — черные дыры подобного размера испарились бы моментально и изучать было бы нечего.

Более-менее стабильные черные дыры начинаются где-то от 1/1000 массы Земли.
Аноним 11/03/19 Пнд 04:00:31 469106359
>>464488 (OP)
А вот возник вопрос, пока смотрел как дыры сливаются.

А как это происходит? Мгновенно или какое-то время занимает?
Аноним 11/03/19 Пнд 04:03:25 469107360
Аноним 11/03/19 Пнд 12:43:12 469180361
blackholemerger.jpg (102Кб, 527x615)
527x615
>>469106
Не мгновенно, но очень быстро, слева вверху же указано время. Начальное сближение может быть сколь угодно долгим, но в конце все происходит за доли секунды.

Чем ближе находятся тела, тем мощнее они излучают гравитационные волны, и тем быстрее они теряют энергию и падают друг на друга. Мощность излучения и скорость сближения пропорциональны кубу расстояния и очень быстро нарастают, в видео даже замедлить пришлось последние микросекунды.

В видео еще упрощенно показано искажение и соединение горизонтов событий, на самом деле оно тоже не мгновенно происходит. Там они должны быть не просто полусферической формы, а скорее в форме этаких запятых, где узкие части направлены друг на друга. Вершины удлинений касаются, сливаются, и перемычка быстро утолщается, пока черные дыры полностью не сольются в форму как на видео.
Аноним 11/03/19 Пнд 15:53:04 469203362
>>469180
Если дыры могут слиться, возможен ли обратный процесс? Сингулярность разделяется на две части и образуются два горизонта событий? По CPT симметрии такое вроде должно быть возможным, не?
Аноним 11/03/19 Пнд 17:25:00 469207363
>>469203
>По CPT симметрии такое вроде должно быть возможным
Зато второй закон термодинамики (а у ЧД охуенно огромная энтропия, причем энтропия итоговой дыры больше энтропии двух исходных) это запрещает for all practical purposes.
Аноним 11/03/19 Пнд 22:32:37 469279364
>>466185
>был ли полет Бурана чем-то охуенным и выдающимся с технической точки зрения?
И был и не был.
Был - потому что первый сверхтяж на водороде, а СССР отставал в криогенных технологиях лет на двадцать, если не больше.
Не был - потому что для СССР этот проект был без реальных задач но с огромными затратами, прост наверху решили "сделойте такую же йобу, чтоб не стыдно перед буржуями".
Задачи Шаттла под которые выбивалось финансирование - обслуживание Хаббла и спутников шпионов сканировавших Атлантику, у СССР этих девайсов не было а сама экономика была из говна и палок.
Аноним 12/03/19 Втр 13:48:43 469402365
Почему нельзя просто выстреливать металлы и материалы на орбиту из пушки и уже там собирать всякие спутники и прочее.
Аноним 12/03/19 Втр 14:26:15 469406366
Анон, доставь вебку из какой-то серии "Expanse", там где полет молодого Манео через кольца Сатурна. Я знаю у тебя есть.
Аноним 12/03/19 Втр 14:33:23 469407367
>>469402
>Почему нельзя просто выстреливать металлы и материалы на орбиту из пушки и уже там собирать всякие спутники и прочее.

для этого на орбите придется построить завод, доменную печь, литейный цех и т. д.
Аноним 12/03/19 Втр 16:00:44 469423368
>>469402
1) Нужно пиздец какое ускорение, чтобы твой полезный груз запульнуть на орбиту, преодолев притяжение Земли и сопротивление атмосферы. Редкий груз такое вообще выдержит.
2) Нельзя вывести что-то на орбиту одномоментным ускорением. Или ты там его ловишь (компенсируя момент движком), или оно ебнется обратно на Землю. Перспектива так себе.
3) Как уже сказал >>469407, нужно будет создать практически полный производственный цикл на орбите. Мало того, что это стоит просто дохуя, кто все это будет обслуживать и поддерживать? На Земле, если у тебя ломается станок, приходит кузьмич, ну или приезжает на экскаваторе, и чинит. Кто на орбите будет ремонтировать?
4) Ну и плюс нужно разработать и отладить все технологии заново, учитывая отсутствие силы тяжести и отсутствие атмосферы (а значит, простого дыхания для обслуги, охлаждения конвекцией и источника кислорода для реакций горения). При том, что масштабные эксперименты в космосе дороги и небезопасны, потому что в случае проблем, оперативно устранять последствия затруднительно.
Аноним 12/03/19 Втр 19:34:29 469464369
> По его словам, уже «сейчас [генеральный директор госкорпорации] Дмитрий Рогозин дал указание начать разработку квантового двигателя».
>
> Бывший директор «Воронежского механического завода» (ВМЗ) Георгий Костин отметил, что в перспективе «квантовый двигатель» придет на смену жидкостному ракетному двигателю.
>
> Советник ракетно-космической корпорации «Энергия», министр общего машиностроения СССР (в 1983–1991 годах), Герой Социалистического Труда Олег Бакланов заявил, что «фундаментальная наука проникла в природу гравитации и антигравитации, квантованную структуру пространства. Это позволяет создавать нереактивные, не требующие химического топлива двигатели для космоса».
>
> По словам автора «квантового двигателя» Владимира Леонова, по космическому пространству «разлита» колоссальная энергия «в виде глобального электромагнитного поля с очень мелкой дискретностью (квантованностью), о котором ранее ничего не было известно». «Это глобальное поле открыто мной в 1996 году как пятая фундаментальная сила (суперсила) в виде сверхсильного электромагнитного взаимодействия (СЭВ). Его носителем является квант пространства-времени (квантон), размеры которого на десять порядков меньше атомного ядра, но он концентрирует энергию, намного превышающую ядерную», — говорит Леонов.
>
> Он заявляет, что «квантовый двигатель отталкивается от глобального поля СЭВ за счет деформации в нужном направлении (искривления по Эйнштейну) квантованного пространства-времени, создавая искусственною силу тяготения (тяги)».

Как тебе такое, Илон Маск?
Аноним 12/03/19 Втр 19:48:13 469472370
Аноним 12/03/19 Втр 20:06:17 469481371
щас смотрел видео про космическую станцию и там вместе живут и мужчины и женщины.
Возник следующий вопрос: ебуться ли они между собой или дрочат? Они же там продолжительное время находятся, все здоровые и в сексуально-активном возрасте. Понятно, что работа у них сверх-ответственная и малейшая невнимательность может привести к плачевному исходу, но неужели они никак не выпускают пар? Что же там происходит?
Аноним 12/03/19 Втр 21:43:13 469533372
>>469472
Хорошо, вопрос такой: какие научные теории могут стать базисом для Роскосмоса при разработке квантового двигателя?
Аноним 12/03/19 Втр 21:49:38 469539373
>>469533
никакие, это бред сумасшедшего
более того, аналогичный антинаучный двигатель роскос уже запускал на орбиту
Аноним 12/03/19 Втр 21:55:01 469547374
>>469539
Хуясе. И как успехи? Ссылка есть?
Аноним 12/03/19 Втр 22:12:51 469557375
>>469547
ищи двигатель гравицапа если сильно интересно, он точно так неофициально назывался
сейчас все результаты поиска загажены ем-драйвом и прочим, мне впадлу серьезно искать
Аноним 13/03/19 Срд 00:40:14 469586376
Допустим, космический аппарат находится на орбите Солнца. Если включить ракетный двигатель выхлопом в сторону движения по орбите, и сбросить таким образом орбитальную скорость до 0, аппарат просто грохнется на Солнце?
Аноним 13/03/19 Срд 00:40:44 469587377
>>469586
> на орбите Солнца
вокруг Солнца
Фикс
Аноним 13/03/19 Срд 00:42:30 469588378
>>469586
Да. Правда, на солнечных орбитах (если не в районе Нептуна какого-нибудь) такие скорости, что это малореально.
Аноним 13/03/19 Срд 00:44:45 469589379
>>469588
Но были ли какие-то зонды, которые прямо на Солнце запуляли?
Аноним 13/03/19 Срд 00:47:45 469590380
>>469589
Сомневаюсь. Хотя что-то пробегало про какой-то исследовательский аппарат (то ли планируют, то ли уже запустили), который будет подлетать к Солнцу ближе, чем Меркурий. Но в любом случае это делается не так, как ты предлагаешь, а гравитационными маневрами. Пушо дешевле.
Аноним 13/03/19 Срд 01:00:01 469593381
Аноним 13/03/19 Срд 01:05:57 469594382
Допустим какой-то мегазлодей захотел спровоцировать солнце на мощный всплеск излучения с целью уничтожить жизнь на Земле. Как это теоретически можно сделать?
Аноним 13/03/19 Срд 01:10:51 469595383
>>469594
Постом выше выяснилось, что на Солнце даже не уронить ничего, даже если сильно захотеть, а ты про такое спрашиваешь))
Аноним 13/03/19 Срд 01:12:23 469596384
>>469595
А если облучать его сверхмощным рентгеновским лазером? Или выстрелить черной дырой?
Аноним 13/03/19 Срд 01:15:17 469597385
Аноним 13/03/19 Срд 01:15:58 469598386
>>469596
Как это теоретически можно сделать?))
Аноним 13/03/19 Срд 01:19:23 469599387
>>469598
С помощью гигантского лазера или ускорителя частиц разумеется.
Аноним 13/03/19 Срд 01:58:41 469600388
>>469595
Уронить можно, гравитационными маневрами. Другое дело, что даже если на Солнце уронить всю Землю, ему ровно нихуя не будет.
Аноним 13/03/19 Срд 02:01:48 469602389
>>469597
Ого, там не просто "ближе Меркурия", а в разы ближе. Ну, собственно, это подтверждает, что уронить можно: эта хуета уже не так и далеко от попадания в Солнце будет.
Аноним 13/03/19 Срд 02:07:19 469605390
>>469600
> даже если на Солнце уронить всю Землю, ему ровно нихуя не будет.
Оно взорвется.
Аноним 13/03/19 Срд 02:08:32 469606391
>>469605
Онемы пересмотрел?
Аноним 13/03/19 Срд 02:11:33 469607392
Аноним 13/03/19 Срд 07:09:22 469616393
Аноним 13/03/19 Срд 13:03:03 469694394
>>469616
Вообще полезный спутник, со всех сторон.
Аноним 13/03/19 Срд 22:12:37 469940395
Что, если на первую ступень прикрутить вертолётные лопасти для посадки?
Аноним 13/03/19 Срд 22:13:38 469942396
Аноним 13/03/19 Срд 22:17:39 469944397
>>469942
А если всё-таки? Вдобавок не надо будет никуда возить, долетит своим ходом.
Аноним 13/03/19 Срд 22:20:28 469946398
Аноним 13/03/19 Срд 22:29:34 469948399
image.png (257Кб, 600x348)
600x348
>>469940
гугли rotary rocket
хуета
Аноним 13/03/19 Срд 22:33:27 469949400
>>469948
Они взлетать вроде хотели. А приземлятся можно на авторотации.
Аноним 14/03/19 Чтв 00:35:37 469959401
Чем дальше объект, тем дольше идет свет от него, так? Мы видим эти объекты такими какими они были сотни, миллионы и даже миллиарды лет назад, так?
Почему тогда на самом горизонте видимой вселенной мы не наблюдаем момента зарождения вселенной?
Аноним 14/03/19 Чтв 00:49:25 469961402
>>469959
>Чем дальше объект, тем дольше идет свет от него, так? Мы видим эти объекты такими какими они были сотни, миллионы и даже миллиарды лет назад, так?
Да.
>Почему тогда на самом горизонте видимой вселенной мы не наблюдаем момента зарождения вселенной?
Потому что изначально Вселенная была непрозрачна для света: слишком много слишком горячего вещества. Плюс, то что мы видим, видим мы хуево. Во-первых, расстояние пиздец огромное: различить объекты мельче галактик (причем ярких) вообще проблематично. Во-вторых, для далеких объектов красное смещение становится очень большим.
Аноним 14/03/19 Чтв 00:49:49 469962403
>>469959
Потому что сначала вселенная была непрозрачной. Самое раннее, что можно увидеть - реликтовое излучение, которое появилось через 400000 лет после БВ.
Аноним 14/03/19 Чтв 01:07:19 469964404
Почему в нашем ближайшем нет квазаров - до всех миллиарды светолет? Со временем они успокаиваются, и поэтому в современной Вселенной их почти нет? Или "на отдалении" они тоже редко встречаются и от одного до другого обычно очень далеко?
Аноним 14/03/19 Чтв 02:23:45 469969405
>>469964
Потухли жи, те что далеко тоже на данный момент потухли, но мы их до сих пор видим светящимися из-за конечности скорости света.
Аноним 14/03/19 Чтв 03:38:55 469976406
будущее россии.jpg (52Кб, 612x546)
612x546
Аноним 14/03/19 Чтв 03:41:31 469977407
1317449868797.jpg (181Кб, 600x415)
600x415
>>469464
Это откуда, ссылки то блять где? Уёбище ты лесное.
Аноним 14/03/19 Чтв 03:48:23 469978408
даблфейспалм1.jpg (44Кб, 600x530)
600x530
>>469616
По началу вроде ничего особенного. Но когда читаешь раздел "Критика", становится понятно, что за "академики" и "инженеры" возглавляют российскую науку. Пиздец... просто пиздец...
Аноним 14/03/19 Чтв 07:52:23 469984409
Что космонавт увидит в центре черной дыры; если дыра большая его же сразу не порвёт, когда он пройдёт через горизонт событий.
Аноним 14/03/19 Чтв 08:44:41 469993410
>>469984
Ничего не увидит, в черной дыре невозможно получить какую-либо информацию с большей глубины. Все направления ведут только глубже вниз, к сингулярности, у света нет возможности как-то вернуться назад и донести тебе информацию о том, что находится ниже.

Да и вообще никакого зрелища не будет, если космонавт свободно падает в сверхмассивную черную дыру, то он даже не заметит собственно момент пересечения горизонта. Вплоть до самой спагеттификации и разборки на атомы ему будет казаться, что горизонт событий все еще находится на каком-то расстоянии перед ним, и что чернота горизонта заполняет менее половины неба.

По ссылке https://jila.colorado.edu/~ajsh/insidebh/schw.html есть хорошее объяснение и видео с симуляцией.

https://vimeo.com/8723702
https://vimeo.com/8818891
Аноним 14/03/19 Чтв 09:17:18 469995411
Как известно вращение Земли замедляется и Луна отдаляется. Если расчеты с какой скорость вращалась Земля, после того как они более мене сформировались с Луной?
Аноним 14/03/19 Чтв 09:31:40 469999412
>>469995
Точно не известно, но обычно считается, что часов 5-6.
Аноним 14/03/19 Чтв 13:01:15 470040413
14778204099140.jpg (44Кб, 500x500)
500x500
А скажите ка мне, вот подтвердили наличие гравитационных волн.
Гравитон безмассовая частица со спином, имеет поляризацию.
Можно построить квантовый усилитель гравитационных волн тогда, типа лазера?

ГРАЗЕР?

Можно ведь за счет резонанса усилить тогда гравиволны одной поляризации, даже совсем слабые, от слабых источников и ИРЛ наблюдать/изучать тогда гравитацию?
Вот только не ясно, что может быть в качестве "поляризационоого фильтра" для гравиволн.

Как вам идея, ИЛОНЫ МАСКИ, а?

Такой ГРАЗЕР может локально деформировать пространственно-временной континуум в выбранном направлении например и служить мощным оружием и/или движителем в пространстве,так ведь?
Аноним 14/03/19 Чтв 13:10:17 470041414
Аноним 14/03/19 Чтв 13:14:29 470042415
image.png (462Кб, 950x506)
950x506
>>470040
>ГРАЗЕР может локально деформировать пространственно-временной континуум в выбранном направлении например и служить мощным оружием

Дратути
Аноним 14/03/19 Чтв 17:14:43 470093416
Наверняка же можно рассчитывать на то, что когда следующий метеорит с Марса упадёт на Землю, НАСА сможет найти место, откуда он откололся на Марсе, найдя новый кратер сравнением снимков с реканессенс орбитера, так? Это должен быть приличных размеров кратер, чтобы кусок Марса улетел с него.
Аноним 14/03/19 Чтв 17:16:54 470094417
>>470093
Более того, если расставить на Марсе регистраторы землетрясений, то сопоставить время и найти место будет еще проще. Хотяя, я ща подумал, что может много времени пройти с момента откалывания куска Марса до момента его падения на Землю
Аноним 14/03/19 Чтв 17:17:35 470095418
Если завтра на Землю ебнется метеорит, аналогичный по размеру и скорости тому, что вызвал мел-палеогеновое вымирание, какие будут последствия для людишек? Скажем, ебнется в густонаселенном районе прямо на Нью-Йорк, рряяяя. Интересуют и локальные и глобальные последствия.
Аноним 14/03/19 Чтв 17:39:46 470098419
>>470040
Подзагугли хотя бы маняпредположения о длине гв.
После этого предположи кратные размеры резонатора. Даже не вдаваясь, из какого маняматериала он должен быть изготовлен.
Аноним 14/03/19 Чтв 17:44:36 470099420
>>470095
Во-первых, мы о нём, скорее всего узнаем заранее. Челябинский метеорит мы пропустили потому что он был маленький и двигался со стороны солнца. Так что в густонаселённом районе не упадёт точно. Если мы узнаем о нём хотя бы за месяц, то людей эвакуируют, а если хотя бы за несколько дней (а меньше не может быть для такой глыбы), то эвакуируют хотя бы большинство из самых ближних районов (ну и зависит от густонаселённости конечно, в логистику вдаваться лень). Дальше будут прямые последствия, разрушения, цунами, вот это всё, но это всё будет известно заранее, людей, опять же, везде эвакуировали. А вот косвенные последствия гораздо существеннее. Скорее всего похолодает, небо будет закрыто выброшенным пеплом, это сильно скажется на урожайности и на экосистемах по всему миру. Переселение огромного количества людей тоже дело не из приятных, и экономика на такое не рассчитана. Ну так, если примерно почувствовать, то от прямых последствий погибнет ну может быть пара сотен тысяч человек, а может и гораздо меньше, а от косвенных сотни тысяч или даже миллионы. Ну а так, в остальном похуй вообще, цивилизация не пострадает, оправимся лет за 50.
Аноним 14/03/19 Чтв 17:46:05 470100421
>>470094
>может много времени пройти с момента откалывания куска Марса до момента его падения на Землю
Я больше скажу, вот так вот чтобы откололся и сразу прилетел - это крайне маловероятно.
Аноним 14/03/19 Чтв 17:50:32 470101422
>>470099
Где-то (по Дискавери?) еще показывали, что удар такой силы выбросит осколки в ближний космос, которые потом, естественно, упадут обратно. И будет "мелкий метеоритный дождь" по всей планете. Или пиздели?
Аноним 14/03/19 Чтв 17:51:36 470102423
>>470098

Подзагуглил. 10^11 Гц максимум. Так что все ок. Конструкция приемлимых размеров вполне возможна.
Аноним 14/03/19 Чтв 17:51:47 470103424
>>470101
Ну это да, это возможно.
Аноним 14/03/19 Чтв 18:35:15 470109425
>>470099
А разве нельзя об него ебнуть какой-нибудь искусственный спутник или ракету?
Аноним 14/03/19 Чтв 18:50:20 470114426
>>470109
А толку, что ты сделаешь глыбе космического говна весом в хуиллиард тонн?
Аноним 14/03/19 Чтв 18:52:30 470115427
>>470114
Ядерной бимбой расколоть можно
Аноним 14/03/19 Чтв 18:54:24 470116428
>>470114
Траекторию чуть поменять.
Аноним 14/03/19 Чтв 19:22:03 470119429
>>470115
>>470116
Нельзя. Он наши бомбы вообще не заметит. А если и удастся расколоть, то ещё может стать только хуже. Вместо одного предсказуемого камня с хорошо измеренной траекторией будут несколько, всё ещё гигантских, летящих в разные места планеты, плюс ещё куча мелких, и точная траектория их всех будет неизвестна.
Аноним 14/03/19 Чтв 19:25:48 470120430
>>470119
Зависит от размера наверное, если вместо одного такого, какой атмосфера не сотрет, будет 5 которые сотрутся об нее - будет гораздо лучше. А бимбу можно любой мощности сделать, можно и Луну к херам взорвать если захотеть, наверное
Аноним 14/03/19 Чтв 19:36:54 470123431
>>470120
>Зависит от размера наверное
Анон изначально спрашивал про такой, как 65 миллионов лет назад упал, он был около 10 км в диаметре, о таком и речь.

>если вместо одного такого, какой атмосфера не сотрет, будет 5 которые сотрутся об нее - будет гораздо лучше
Да, но такой большой астероид, как мы рассматриваем, даже если расколоть на 5 частей, это всё ещё будут огромные глыбы, которые легко пройдут атмосферу.

>А бимбу можно любой мощности сделать, можно и Луну к херам взорвать если захотеть, наверное
Эм, нет, это абсолютно за гранью наших возможностей. Какую там Луну, о чём ты. Мало того, что бомбу надо сделать, её ещё туда доставить надо. Ну а бомбу чтобы взорвать Луну мы ещё в ближайшие столетия не сможем создать как минимум.
Аноним 14/03/19 Чтв 19:48:45 470124432
>>470123
> это абсолютно за гранью наших возможностей
А я читал, что ядерные бимбы году к 70му СССР и США научились делать практически неограниченной мощности

А доставить на Луну ракетой-носителем можно, как и обычные ядерные бимбы токо покруче
Аноним 14/03/19 Чтв 20:12:37 470131433
>>470124
Бля, чувак, ну что значит неограниченной мощности? Мощность ограничена количеством вещества, которое вступает в реакцию. Да, ты можешь сделать бомбу любой мощности, если соберёшь достаточно дейтерия, лития, урана, что там ещё в них используется. Чтобы взорвать Луну не хватит всех запасов, которые у нас есть, всех бомб, которые когда-либо были созданы. Всего современного ядерного арсенала возможно не хватит, чтобы расколоть даже 10-километровый камушек. Ну какая Луна, ты хоть представляешь себе её масштабы? Бомба для взрыва Луны должна быть сама несколько километров в диаметре.

>А доставить на Луну ракетой-носителем можно, как и обычные ядерные бимбы токо покруче
Ну да блядь, делов-то, всего лишь ракету сделать. Ну так, на несколько миллионов тонн полезной нагрузки. Муск в гараже состряпает.
Аноним 14/03/19 Чтв 20:17:24 470135434
>>470131
> Всего современного ядерного арсенала возможно не хватит, чтобы расколоть даже 10-километровый камушек.
Вот уж в этом сомневаюсь, бомба раз в 100 мощнее царь-бомбы наверняка ее расколет
Аноним 14/03/19 Чтв 20:21:08 470138435
>>470135
Может быть. Но доставить её туда не представляется возможным. Ну и, как я уже говорил, это мало чем поможет, а то и хуже сделает.
Аноним 14/03/19 Чтв 20:29:45 470139436
15519006192580.png (294Кб, 692x395)
692x395
>>470138
> Но доставить её туда не представляется возможным.
Аноним 14/03/19 Чтв 21:35:21 470155437
>>470135
>>470138
Ловите пасту.

> Ну давай прикинем порядок. Энергия гравитационной связи Земли (https://en.wikipedia.org/wiki/Gravitational_binding_energy) около 2 10^32 Дж. Это та энергия, которая нужна чтобы разорвать планету на части. Будем считать, что чтобы она треснула, достаточно 1/50 части этой энергии (хотя этого наверняка слишком мало), то есть 4 10^30 Дж.
>
> Тонна в тротиловом эквиваленте примерно равна 4 10^9 Дж. То есть, нам понадобится около 10^21 тонн в тротиловом эквиваленте.
>
> Царь-бомба имела массу порядка 25 тонн и выдала мощность порядка 50 Мт. То есть, для термоядерного заряда примерная удельная мощность - 2 10^6 тонн в эквиваленте на тонну заряда. Таким образом, гипотетическая термоядерная бомба, способная расколоть Землю, будет весить примерно 10^21 / 2 10^6 = 5 10^14 тонн.
>
> Масса Земли - 6 10^21 тонн. У нашей бомбы - примерно в 10^7 раз меньше. Значит, если сделать бомбу в виде шара с плотностью как у Земли, то он будет в 200-250 раз (кубический корень) меньше Земли радиусом, то есть с радиусом в 25-30 км.
>
> Чтобы расколоть Землю понадобится термоядерная бомба в виде шара диаметром примерно 50-60 км.

В 100 раз мощнее Царь-бомбы - курам на смех. Собственно, метеорит, упавший 65 миллионов лет назад был в 2 миллиона раз мощнее той бомбы. И хули, он даже тоненькую (менее процента радиуса) земную кору не пробил.
Аноним 14/03/19 Чтв 21:44:33 470160438
>>470155
50 км шар - ого. Я разочаровался в ядерных бомбах;(

На Землю диаметром 12000 км бомба диаметром 50 км - в 240 раз меньше

Но если тот метеорит был 10 км, то бомба, которая его расколет, будет по аналогии 10 / 240 = 41 метр диаметром. Царь-бомба была 8х2 метра. Нарастить раз в 6 - и рванет что надо.
Аноним 14/03/19 Чтв 21:48:17 470163439
>>470160
Ну там гораздо меньшего хватит, потому что метеорит сам по себе имеет крошечное тяготение. То есть энергия гравитационной связи падает/растет гораздо быстрее, чем куб радиуса (не скажу с какой именно скоростью, правда). Но для Луны тяготение уже значительно.
Аноним 14/03/19 Чтв 21:50:32 470165440
Аноним 14/03/19 Чтв 21:50:43 470166441
>>470163
О, значит метеорит вообще на дохера кусков разлетится и все будет чики-пуки
Аноним 14/03/19 Чтв 21:52:48 470167442
>>470165
41х41= 1681
8х2=16
В сто раз, в точности как я и говорил изначально)))
А учитывая слабую гравитационную связь и того меньше
Аноним 14/03/19 Чтв 21:53:29 470168443
>>470167
А, падажжи, это ж куб, тогда да, хировато
Аноним 14/03/19 Чтв 21:59:18 470175444
В любом случае, мы даже царь-бомбу не сможем дальше НОО отправить с существующими ракетами.
Аноним 14/03/19 Чтв 22:00:12 470177445
А ведь еще можно лазером что-нибудь сделать с астероидом - испарять на одной его стороне что-нибудь (газы чтоб вылетали реактивной струёй) и таким образом отклонить орбиту, или бред несу?
Аноним 14/03/19 Чтв 22:02:05 470178446
>>470177
Если у тебя вагон времени, то можно, в теории. На практике - сам понимаешь.
Аноним 14/03/19 Чтв 22:02:47 470179447
Аноним 14/03/19 Чтв 22:18:44 470194448
>>470163
>энергия гравитационной связи падает/растет гораздо быстрее, чем куб радиуса (не скажу с какой именно скоростью, правда)
А, собственно, в той статье есть формула. Она для равномерной плотности, но это не принципиально для порядка, там только коэффициент будет другой. Короче, учитывая, что масса растет как куб радиуса, получаем, что энергия гравитационной связи пропорциональна пятой степени радиуса. Так что для астероида 10 км в диаметре хватит и одной царь-бомбы за глаза.
Аноним 14/03/19 Чтв 22:21:41 470199449
>>470155
>Таким образом, гипотетическая термоядерная бомба, способная расколоть Землю, будет весить примерно 10^21 / 2 10^6 = 5 10^14 тонн
Бля ну охуеть теперь экстраполяция.жпг
Аноним 14/03/19 Чтв 22:35:40 470210450
Почему на первой ступени ракеты-носителя не использовать обычный ЖРД с окислением атмосферным воздухом? Нахуя на земле сжигать драгоценнейший жидкий кислород из баков? Разве так мы не сэкономим на массе? На атмосферном участке полета нам тяга не так важна: тут задача лишь в том чтобы выйти в стратосферу, а там уж горизонтально будем разгоняться как положено.
Аноним 14/03/19 Чтв 22:47:27 470217451
Аноним 14/03/19 Чтв 22:50:36 470218452
Как принять участие в лунном заговоре?
Аноним 14/03/19 Чтв 23:35:40 470237453
>>470218
Родиться пиндосом в 20-40-е годы.
Аноним 15/03/19 Птн 00:18:44 470245454
Меня очень сильно пугают чёрные дыры, да и вообще осознание того, что мы живём на шаре который летит хуй знает куда. Помню один раз накатил SpaceEngine и сам безграничный космос меня просто пугал похлеще любого хоррора. В Интерстелларе, когда показывали чёрную дыру, мне просто становилось не по себе. Я один тут такой ебанутый?
Аноним 15/03/19 Птн 00:27:17 470246455
>>470245
Тут когда-то был целый тред таких ебанутых, не знаю, что с ним сейчас, надеюсь потёрли.
Аноним 15/03/19 Птн 01:25:38 470255456
image.jpeg (1107Кб, 2208x1242)
2208x1242
Вот еще про бимбы и астероиды
Аноним 15/03/19 Птн 02:14:03 470263457
>>470210

Не используют, потому что таких долбоебов нет.

Во-первых, любые воздушно-реактивные двигатели обладают ничтожной удельной тягой по сравнению с ракетными, в десятки раз ниже. Соответственно, чтобы ракета имела тяговооруженность больше 1 и вообще могла хотя бы подняться со стартового стола, устанавливаемые на нее двигатели уже получаются размером с саму ракету.

Во-вторых, любой воздушно-реактивный двигатель максимально эффективен только на малом диапазоне скоростей. Турбореактивные хорошо работают в начале, но начинают глохнуть уже после двух скоростей звука, прямоточные ВРД лучше работают на нескольких скоростях звука, но стоя на месте не выдают вообще никакой тяги и для старта с земли абсолютно непригодны, выше 5 махов нужно сверхзвуковое горение, с которым пока сильные проблемы и тоже нужен свой двигатель.

В-третьих, воздух при подъеме очень быстро заканчивается. Первая ступень того же фалкона сбрасывается на высоте 60+ километров и скорости ~5 махов, на такой высоте ни один современный двигатель не работает.

Так что либо ты изобретаешь какие-то сказочные двигатели, которые одновременно и с места могут выдавать невиданную тягу, и на гиперзвуке работать, и весить немного, либо твоя ракета получается увешанной целыми батареями разнорежимных двигателей, либо ты не занимаешься долбоебизмом и окислитель с собой возишь, как все нормальные люди.
Аноним 15/03/19 Птн 17:04:37 470401458
>>470160
50км - это размер заряда.
Размер заряда кузькиной мамки был ~0,5-1,0м.
К 50км заряду добавь ещё всю инициирующую обвязку. При таких масштабах она легко приблизится к размерам Луняши.
Аноним 15/03/19 Птн 17:57:08 470415459
>>470401
Там при расчете бралась полная масса матери, с инициируещей обвязкой аэродинамическими конструкциями и т. д. В оригинале вроде даже был свинцовый балласт как замена урану, который бы добавил мощи. Так что норм. Хотя боньба таких размеров потребовала бы еще всякой несущей инженерной хуйни будь здоров, это да.
Аноним 15/03/19 Птн 20:31:35 470448460
>>470175
Запулить бомбу, потом разгонную ступень для неё отдельно, состыковать. Пиздец вы тут маньки.
Аноним 15/03/19 Птн 21:17:39 470462461
>>470448
Это в теории (или в Кербале) все возможно. На практике нужны годы разработок, чтобы не получился очередной роскосмос на орбите.
Аноним 15/03/19 Птн 23:31:58 470518462
Что если темная материя это триллиарды черных дыр, которых мы не видим только из-за того что в данный момент времени ничто не взаимодействует с ними?
Аноним 15/03/19 Птн 23:32:21 470519463
Аноним 15/03/19 Птн 23:35:30 470520464
>>470518
Гугли планковские черные дыры. Короче, есть такая версия. Только там не "в данный момент", а вероятность с ними провзаимодействовать вообще ничтожная, меньше чем с нейтрино.
Аноним 15/03/19 Птн 23:57:47 470535465
>>470520
Ну, темная материя же влияет на расширение, планковские черные дыры не способны на такое?
Я имею ввиду что если допустим все межзвездное пространство заполнено черными дырами, а звезды образовались только потому что рядом таких скоплений не было, а современные звезды с ними нашли равновесие за столь долгое время. И допустим, пошлем мы вояджер за пределы солнечной системы, а тот бац и упадет в черную дыру. И так по всем направлениям, и мы не сможем улететь из нашей вселенной.
Аноним 16/03/19 Суб 00:26:22 470553466
14087360461270.jpg (120Кб, 662x900)
662x900
Если в космосе окажется что-то вроде ракеты союз полностью заправленной, куда можно закинуть аппарат весом например в 3 тонны за 10 лет полета?
Аноним 16/03/19 Суб 00:47:45 470573467
>>470518
Ну это один из теоретических вариантов, подразумевающий барионный ориджин темной материи. Там не только черные дыры, но много помимо них вполне привычной формы вещества в кандидаты. Просто по уравнениям эволюции вселенной не сходится.
Аноним 16/03/19 Суб 00:53:13 470575468
>>470535
>Ну, темная материя же влияет на расширение
На расширение влияет темная энергия. Это "еще более темная" хуйня. По теориям (тут ОТО ни при чем), вакуум или пространство само по себе обладает таким свойством.
>Я имею ввиду что если допустим все межзвездное пространство заполнено черными дырами, а звезды образовались только потому что рядом таких скоплений не было, а современные звезды с ними нашли равновесие за столь долгое время. И допустим, пошлем мы вояджер за пределы солнечной системы, а тот бац и упадет в черную дыру. И так по всем направлениям, и мы не сможем улететь из нашей вселенной.
Массивных черных дыр не может быть так много, это не увязывается с наблюдениями.

Планковские черные дыры, как раз, имеют малую массу - планковскую (~20 миллионных грамма). А поскольку гравитация вообще-то слабая штука (тебя сильно притягивает табуретка?), то такая дыра тянет очень-очень слабо, да и радиус у нее милипиздрический. Такая дыра сквозь Солнце пролетит и ни с чем не прореагирует.
Аноним 16/03/19 Суб 02:25:36 470592469
>>470575
>тебя сильно притягивает табуретка?
Ускорение свободного падения зависит не только от массы, но и от плотности, поэтому аналогия с табуреткой некорректна. Сжатая до планковских размеров табуретка будет тянуть сильнее.
Другое дело, что у микрочёрной дыры малое сечение захвата и расталкивающее падающее вещество давление излучения хокинга, да и срок жизни у них короткий.
Аноним 16/03/19 Суб 02:29:03 470593470
>>470592
>Ускорение свободного падения зависит не только от массы, но и от плотности
Нет, не зависит. Посмотри на закон всемирного тяготения: сила зависит только от массы притягивающего вещества и от растояния. Все.
Аноним 16/03/19 Суб 09:23:00 470606471
>>470593
>Посмотри на закон всемирного тяготения
Нет, извини, в нашем треде смотрят больше на уравнения Эйнштейна.
Аноним 16/03/19 Суб 09:45:11 470607472
>>470263
>воздушно-реактивные двигатели обладают ничтожной удельной тягой
Нихуя себе. По ходу дела сверхзвуковых стратосферных самолетов не существует, это газетная утка, а миг-25 просто троллинг и фейк.
>устанавливаемые на нее двигатели уже получаются размером с саму ракету
Уравнения приведи, а не сопливые вскукареки(хуй знает, как ты считаешь размер движка?)
Аноним 16/03/19 Суб 10:04:10 470608473
изображение.png (301Кб, 640x480)
640x480
Аноним 16/03/19 Суб 10:45:27 470610474
>>470593
Очевидно, что >>470592-анон имел в виду g на поверхности тела. А оно растёт при фиксированой массе тела и увеличении его плотности, просто потому что растояние от поверхности до центра падает.

>>470606
А? Я ненастоящий сварщик, но вроде именно в этом вопросе у Эйнштейна то же, что у Ньютона.
Аноним 16/03/19 Суб 11:32:10 470612475
>>464488 (OP)
В центре Земли находится частичка чёрной дыры, такой же как в центре галактики. Образовалась материя в чёрной дыре - в результате взрыва сверхновой галактики. Влияние этой материи на материю объективного мира с нашими уровнём развития органов осязания и объективного понимания - воспринимается как чёрная дыра. Хотя ни дырой и ни чёрной она не является.
Где я что не правильно понял?
Аноним 16/03/19 Суб 11:36:36 470613476
>>470612
>Где я что не правильно понял?
Я бля даже не знаю, что тут ответить... Ты где всю эту наркоманию вообще взял?
Аноним 16/03/19 Суб 11:37:26 470614477
>>470607
Двигатель Д-30Ф6, устанавливаемый на МиГ-31 — масса 2416 кг, максимальная тяга на форсаже 152 кН, удельная тяга 6,41.
Мерлин-1Д, устанавливаемый на Фалкон 9 — масса 470 кг, максимальная тяга 845 кН, удельная тяга 180.
Аноним 16/03/19 Суб 11:38:10 470615478
>>470613
Ладно - как ты объяснишь наличие оче оче высокой температуры в центе Земли? Как у Солнца - термоядерный взрыв водорода?
16/03/19 Суб 11:48:52 470616479
>>470615
>наличие оче оче высокой температуры в центе Земли
Гравитационное сжатие и постепенный распад радиоактивных элементов, входивших в состав протопланетного облака, из которого появилась солнечная система. Эти радиоактивные элементы все тяжёлые и поэтому утонули в ядро Земли. (В первую очередь, наверное, уран-235? Тут я хз.)

>у Солнца - термоядерный взрыв водорода
Не взрыв, а относительно спокойное горение.
Аноним 16/03/19 Суб 11:50:56 470617480
>>470615
Нет, термоядерных реакций там нет, планета с самого начала была горячей за счёт самого процесса планетообразования и перемешивания внутренностей, и затем начала остывать, начиная с внешних слоёв, и всё ещё остывает. Луна несколько замедляет остывание своими приливными силами.
Аноним 16/03/19 Суб 11:55:30 470620481
>>470616
>протопланетного облака
почему внутри планеты земля огненный шар?
ты понимаешь какие температуры должны быть чтобы двигать континенты и вулканы ебашить...
>>470617
>и затем начала остывать
чтобы так долго остывать вещества в планете нужно в тысячу раз больше по объёму.
я напомню планете 5 лярдов лет и вулканы до сих пор хуярят только в путь.
Аноним 16/03/19 Суб 13:01:43 470629482
>>470620
>почему внутри планеты земля огненный шар?
Не огненный шар, а раскалённое железо.

>чтобы так долго остывать вещества в планете нужно в тысячу раз больше по объёму.
Расчёты-то выложи свои, а то мне интересно, откуда ты вообще исходные данные брал.
Аноним 16/03/19 Суб 13:04:37 470631483
>>470616
>уран-235
Уран 238 тяжелее, чем 235, а значит в центре Земли был бы именно он. Но 238 не распадается, а значит Земля внутри не была бы горячей.
Шах и мат, шаросектанты.
Аноним 16/03/19 Суб 13:05:44 470634484
>>470620
>чтобы так долго остывать вещества в планете нужно в тысячу раз больше по объёму.
Ты примерно почувствовал?
Аноним 16/03/19 Суб 17:25:07 470667485
>>470634
Прошлый год: 674 тысячных. Этот год: 704 тысячных. Да, извини, совсем другая картина, что это я.
Аноним 16/03/19 Суб 17:27:43 470668486
>>470667
Простите, поцоны, промазол.
Аноним 16/03/19 Суб 18:49:58 470674487
>>470614
А теперь с массой окислителя, плез.
Аноним 16/03/19 Суб 19:12:02 470675488
>>470674
Турбина будет работать до 20 км, и всё. Ракета до туда долетает за секунды.
Аноним 16/03/19 Суб 19:15:21 470676489
Аноним 16/03/19 Суб 19:22:31 470679490
>>470676
До сюда минут за 20 после обнаружения, до метро добежать успеешь.
Аноним 16/03/19 Суб 20:26:16 470690491
>>470679
Если это не питерское метро то можно не потеть перед смертью.
Аноним 17/03/19 Вск 04:02:44 470762492
>>470612
Пиздец. Это нечто гениальное. Анон, ты не из будущего к нам пришёл?
Аноним 17/03/19 Вск 16:55:16 470931493
Если объект находится на орбите планеты и достигает второй космической (и превышает её), то он в любом случае покинет орбиту этой планеты?
Аноним 17/03/19 Вск 17:05:49 470938494
>>470931
Да, при условии что он разгоняется прямо по курсу, ну или куда-то ещё, но не в планету или её атмосферу.
Аноним 17/03/19 Вск 17:11:14 470941495
>>470938
А на орбите на скорости более, чем вторая, как можно остаться? Только корректируя курс двигателями (говорим о космическом корабле)?
Аноним 17/03/19 Вск 17:24:31 470945496
>>470941
Ага. Нужно будет постоянно жарить двигателями в направлении планеты (или строже говоря, барицентра), имитируя таким образом более сильное гравитационное притяжение.
Аноним 17/03/19 Вск 17:28:20 470946497
Аноним 17/03/19 Вск 17:47:07 470955498
>>470941
Ну, ещё можно привязаться верёвочкой, или удерживаться магнитным полем (частицы в радиационных поясах так и делают).
Аноним 17/03/19 Вск 20:16:47 471006499
Аноним 17/03/19 Вск 20:25:09 471015500
Аноним 17/03/19 Вск 20:27:58 471016501
>>471015
Так сильно что аж засветку создает?
Аноним 17/03/19 Вск 21:02:03 471029502
>>471016
на таймлапсе небось
Аноним 17/03/19 Вск 21:09:23 471032503
squidfishingboa[...].jpg (68Кб, 720x480)
720x480
Аноним 17/03/19 Вск 21:53:32 471043504
>>471032
Какой же улов тупой, пиздец просто.
Аноним 17/03/19 Вск 22:36:07 471057505
image.jpeg (64Кб, 365x228)
365x228
>>464488 (OP)
Если можно видеть галактику, изображение которой шло до земли 10 миллиардов световых лет, значит можно посмотреть «в спину» этому изображению, или я что-то не так понимайу
Аноним 18/03/19 Пнд 00:00:54 471079506
>>471057
Что значит "в спину"? Ну, ты видишь галактику такой, какой она была 10 млрд. лет назад, такая себе бесплатная машина времени над головой.
Аноним 18/03/19 Пнд 00:34:31 471088507
>>471057
>значит
Не улавливаю логику.
Аноним 18/03/19 Пнд 01:53:55 471118508
Почему лед с мелких комет и астероидов не сублимируется и не улетает в космос? У них же нет гравитации, вода должна свободно улетать, разве нет?
Аноним 18/03/19 Пнд 08:36:43 471141509
>>470620
Пиздец дегенерат. И бывают же такие.
Аноним 18/03/19 Пнд 09:32:38 471149510
>>471057
Если ты имеешь в виду посмотреть в противоположную галактике сторону и увидеть как она выглядит с другой стороны то это не возможжно. свет удаляется со световой скоростью, и догнать своими глазками ты его никак не сможешь..
Аноним 18/03/19 Пнд 11:45:18 471191511
>>471118
С чего бы ему сублимироваться, когда он холодный и твердый? При нагреве сублимируется, если тело вблизи Солнца пролетает, а без нагрева лед стабилен.

>У них же нет гравитации
Гравитация есть у всего. Как, по-твоему, эти самые кометы и астероиды образовались изначально?
Аноним 18/03/19 Пнд 13:19:00 471209512
potw1208a.jpg (58Кб, 700x664)
700x664
У меня тупой вопрос.

Когда Эта Киля взорвется как гиперновая, то это как-то повлияет на Землю? Грубо говорят до нас "достанет" гамма?
Аноним 18/03/19 Пнд 13:53:33 471222513
Аноним 18/03/19 Пнд 14:35:45 471240514
>>471222
>Если направлена - то пиздей

Нас сожжет насмерть?
Аноним 18/03/19 Пнд 15:40:11 471250515
>>471240
> ...прямое попадание гамма-всплеска может привести к катастрофическим повреждениям и серьезному массовому вымиранию. Расчеты показывают, что накопленная энергия такого гамма-всплеска, поразившего земную атмосферу, будет эквивалентна одной килотонне тринитротолуола на квадратный километр по всему полушарию, обращенному к звезде, причем ионизирующее излучение будет в десять раз превышать смертельную дозу облучения всего организма[114].

Оттуда же.
Аноним 18/03/19 Пнд 16:01:18 471254516
>>471250
А в другом полушарии будет похуй?
Аноним 18/03/19 Пнд 16:46:11 471267517
>>471254
Не совсем. Такой пиздец приведет к глобальному изменению климата. Первые дни будут ураганные ветры по всей планете, как я понимаю. Возможно, атмосферу вообще сильно проколбасит, но с другой стороны химсостав поменяться не должен (ну озон там распадется, будет хорошо жарить ультрафиолетом сколько-то там лет). Но по сравнению с "затронутым полушарием" да, легко отделаются.

Алсо, там последствия неравномерны для полушария неудачников. Величина гамма-облучения будет убывать как косинус угла источника в небе. У кого он будет 90 градусов (зенит), те прожарятся особенно хорошо.
Аноним 18/03/19 Пнд 17:11:23 471270518
>>471254
Смотри последствия всяких пермских или силлурийских вымираний.
Аноним 18/03/19 Пнд 17:37:22 471275519
>>471250
Так одна килотонна на квадратный километр это же вообще ничто, на квадратный метр это всего-то 1 грамм тротила или 4200 джоулей, Солнце в зените столько заливает за четыре секунды. Гамма-всплеск тоже не моментально происходит, так что с чисто энергетической точки зрения эффект будет тот же, как если бы на несколько секунд зажглось и погасло второе Солнце.

Стоящие прямо под лучами возможно и подохнут от радиации, да, но людям под землей или на другой стороне планеты должно быть полностью похуй.
Аноним 18/03/19 Пнд 17:54:33 471278520
>>471275
>одна килотонна на квадратный километр это же вообще ничто, на квадратный метр это всего-то 1 грамм тротила или 4200 джоулей
Три нолика потерял. Это один килограмм на квадрат.

Можно прикинуть через площадь Земли. У меня получилось 250 000 мегатонн в эквиваленте. Впрочем да, ничего такого ужасного. Для сравнения, чиксулубский метеорит оценивается в 100 000 000 тератонн, в 400 раз больше.
Аноним 18/03/19 Пнд 17:55:04 471279521
>>471278
> 100 000 000 мегатонн
быстрофикс
Аноним 18/03/19 Пнд 19:45:37 471312522
106067183fireba[...].png (152Кб, 1335x1059)
1335x1059
Че за дичь?


В декабре 2018 года произошёл крупнейший взрыв метеорита после случая в Челябинске в 2013 году.

Метеорит при разрушении об атмосферу выделил энергии в 10 раз больше, чем атомная бомба, сброшенная на Хиросиму. Однако это произошло вдали от людей, поэтому явление осталось практически незамеченным. Об этом рассказал «Би-би-си» сотрудник планетарной защиты NASA Линдли Джонсон.

https://tjournal.ru/science/90583-nasa-rasskazalo-o-krupneyshem-posle-chelyabinska-vzryve-meteorita-na-kamchatke-kotoryy-nikto-ne-zametil
https://www.bbc.com/news/science-environment-47607696

Я могу поверить, что кроме швятых сотрудников НАСА никто не наблюдает ни за космосом, ни за атмосферными явлениями, но чтобы "никто не заметил взрыва мощностью 10 Хиросим"? Современные средства обнаружения ракетных пусков позволяют фиксировать старт отдельных ракет по огневому следу, чуть ли не крылатые ракеты уже видят, а метео спутники фиксируют изменения температур в пределах десятых долей одного градуса. ЧТО ЗА ДИЧЬ?
Аноним 18/03/19 Пнд 20:41:02 471324523
Неужели Вселенная такая маленькая и виде тора??

Аноним 18/03/19 Пнд 20:41:19 471325524
Аноним 18/03/19 Пнд 20:43:36 471327525
За пределами Метагалактики, возможны ли объекты(в виде других Метагалактик или других структур) которые произошли не от Большого взрыва?
Аноним 18/03/19 Пнд 21:23:38 471335526
Аноним 18/03/19 Пнд 22:18:49 471351527
>>471327
Один хуй. Если они и есть, мы с ними никак не взаимодействуем. Ну или так своеобразно взаимодействуем, что все эти взаимодействия замечательно описываются внутри нашего мира.
Аноним 18/03/19 Пнд 23:51:53 471364528
image.png (17Кб, 550x466)
550x466
image.png (20Кб, 966x772)
966x772
Ещё тупых вопросов.
1. Тело выведенное на эллиптическую орбиту со временем там и останется, или орбита будет стремиться к окружности?
2. Пикрил с красной стрелкой. Если телу на орбите (чёрная) будет предана небольшая скорость в направлении Земли (синий круг) (например, исчисляемая всего десятками метров в секунду), то это тело в конце концов должно же упасть на Землю?
3. А если телу предать скорость в противоположном напревлении от п.2, то тело в какой-то момент придёт к относительно стабильной высоте за счёт гравитации земли?
4. А если как в п.3, но телу на орбите придать не просто скорости, а, например, в десятки раз больше, чем вторая космическая для этой планеты? Как тело уйдёт с орбиты? Ближе к зелёной траектории, или всё же планета будет оказывать и тут существенное влияние и оранжевая траектория ближе к правде?
5. На орбите ориентация космических аппаратов параллельно земной поверхности отсутствует же и остаётся относительно постоянной?
Аноним 19/03/19 Втр 00:20:32 471368529
>>471364
1. На очень больших промежутках времени будет стремиться к окружности, на масштабах тысяч лет и менее — можно считать, что останется такой же, как и была, если тяготение других тел не сместит орбиту.

2 и 3. Ничего никуда не упадет и не улетит, просто чуть-чуть поднимется или опустится перицентр/апоцентр орбиты, будет у тебя орбита не 400×400 км, а, скажем, 390×400 или 400×410.

4. Если скорость именно в десятки раз больше — практически по прямой линии, тело улетит слишком быстро, чтобы планета могла существенно изменить траекторию.

5. Отсутствует. Если хочешь, чтобы одна сторона спутника всегда смотрела на Землю — придавай ему вращение со скоростью 1 оборот вокруг своей оси за 1 виток орбиты. МКС, например, так и вращается.
Аноним 19/03/19 Втр 00:45:35 471370530
1552945456525.jpg (77Кб, 1300x941)
1300x941
1552945456532.jpg (65Кб, 1200x1200)
1200x1200
Это правда, что настоящий спейсачер всегда выберет первый вариант?
Аноним 19/03/19 Втр 01:12:27 471375531
>>471267
Какой ужас. Как спастись от гамма-вспышки?
Аноним 19/03/19 Втр 01:27:12 471378532
>>471364
1. Если не учитывать влияние сторонних тел, приливных сил и всяких тонких эффектов ОТО (то есть, если взять сферическое тело в вакууме), то останется навсегда.
2. Не понял, ты постоянно собираешься жарить в сторону Земли или однократно? Если однократно - перейдет на другую орбиту, но проходящую через точку, где придали ускорение. Если постоянно - то рано или поздно ебнется, но по-моему даже выйдет за первоначальный круг.
3. Аналогично для однократного, для постоянного - улетит нахуй.
4. Оранжевая ближе, но все зависит от точного значения скорости.
5. Она постоянная "относительно звезд". Если хочешь постоянно быть повернутым к Земле пузом, нужно вращаться с той же скоростью, что и круги мотаешь.
Аноним 19/03/19 Втр 01:32:08 471379533
>>471351
тёмная энергия например
Аноним 19/03/19 Втр 01:35:00 471380534
>>471378
>но по-моему даже выйдет за первоначальный круг
Ща подумол, пожалуй, не выйдет. Просто будет снижаться, причем все быстрее и быстрее, пока не ебнется.
Перекат Аноним 19/03/19 Втр 02:31:33 471384535
hubblemain.jpg (182Кб, 1280x720)
1280x720
Аноним 19/03/19 Втр 10:19:05 471434536
>>471312
А кто этот взрыв непосредственно мог напрямую наблюдать? Два с половиной рыбака, вот и все. А соответствующие службы наблюдают.
Аноним 19/03/19 Втр 17:43:30 471524537
Аноним 19/03/19 Втр 17:43:55 471525538
>>471370
Нет, сникерс нажористее
Аноним 20/03/19 Срд 17:47:05 471756539
>>464488 (OP)
Что за срань, эти ваши страпельки?
Аноним 20/03/19 Срд 21:47:55 471845540
Ещё вопрос по орбите. Если орбита спутника наклонена относительно оси Земли, при этом направление движения спутника совпадает с направлением вращения Земли, то что случится, если начать корректировать курс спутника, увеличивая наклон и пройдя таким образом в какой-то момент ось вращения Земли? Спутник будет двигаться в противоположную от направления вращения земли сторону?
Аноним 27/03/19 Срд 22:51:33 473657541
Возможно ли существование целого кольца на орбите планеты? Не кольца из камней, как у Сатурна, а целого монолитного вращающегося кольца.
Примерно чувствую, что такое кольцо существовать не сможет, но не могу объяснить почему:

В принципе, целое монолитное кольцо можно представить состоящим из отдельных близких кусков, летящих по орбите, и тогда кольцо должно быть стабильным.

Еще можно представить, что мы берем кольцо как у сатурна, и начинаем протягивать между отдельными глыбами тонкие нити. На этом этапе ничего не должно произойти. Начинаем постепенно утолщать нити, пока помностью не заполним промежутки между камнями. В какой момент все это обрушится?
Аноним 12/05/19 Вск 19:12:34 483223542
>>473657
Угловая скорость вращения тем больше, чем ближе к планете. Если кольцо Сатурна внезапно сделать целым - его разорвет с внутренней стороны гравитацией, с внешней - центробежной силой, прочность на таких масштабах ни на что не влияет. Очень тонкое кольцо сможет просуществовать некоторое время, но будет неустойчивым по тем же причинам.
Настройки X
Ответить в тред X
15000 [S]
Макс объем: 40Mб, макс кол-во файлов: 4
Кликни/брось файл/ctrl-v
Стикеры X
Избранное / Топ тредов